Chemistry

Pataasin ang iyong marka sa homework at exams ngayon gamit ang Quizwiz!

q15

concept: The molar concentration (molarity) of a chemical species is defined as the number of moles of the species per liter of solution (mol/L).

q5

Keq is given so set up the equation and solve for the missing concentration concept: The equilibrium constant (Kp) of a reaction is the molar ratio of products to reactants when equilibrium is achieved. Each product or reactant must be raised to the power of its stoichiometric coefficient in the equilibrium expression.

q28

they are asking for the magnitude of CHANGE IN TEMPERATURE. You have to find T final and T initial on the graph and subtract T final = 33 degree C T initial = 23 degree C Change in temp = 10 degree C concept: Calorimetry experiments measure the temperature as a reaction progresses. Any change in temperature ΔT during the reaction is found by subtracting the initial temperature Ti at the beginning of the reaction from the final temperature Tf at the end of the reaction.

q34

they are asking what formed Uranium after emission of alpha particle so gave you end product so you have to +4 for mass number, and +2 for atomic number Educational objective: Alpha decay is the emission of an alpha particle (helium nucleus) containing two neutrons, two protons, and no electrons from the parent atom. It results in a daughter nucleus with a mass number that is four less than its parent and an atomic number that is decreased by two."

question 48

they asking about bonding in region 1 so looked at region 1 and it had O and N so picked hydrogen bonding concept: Hydrogen bonding is one of the stronger intramolecular and intermolecular forces. It is a special kind of noncovalent dipole-dipole interaction between dipoles formed when hydrogen is covalently bound to the highly electronegative elements nitrogen, oxygen, or fluorine.

question 51

The side chain of valine is branched (more hydrocarbon units) than the side chain of alanine. Consequently, replacing the larger valine with the smaller alanine in region IV will cause the extent of London dispersion forces (van der Waals interactions) to decrease. concept: London dispersion forces are a type of van der Waals interaction by which molecules exhibit mutual attraction because of instantaneous dipoles induced by momentary distortions in the electron distribution within the molecules. London dispersion forces tend to be more pronounced in larger molecules with a larger, more polarizable electron cloud.

Question 4

Increasing bond strength is inversely related to bond length and acidity. As bond order increases (single->double->triple), bond length decreases, bond strength increases and bond energy increases. concept: Covalent bonds are formed by sharing electrons between atoms through the overlap of atomic orbitals in an end-to-end or side-to-side configuration. Multiple bonds formed by s and p orbitals consist of one σ bond and one or more π bonds, and bond length tends to decrease relative to a single bond with each added π bond.

question 23

they gave the pH and which basically gives us the H+ concentration and then they are asking for OH concentration. So, just do 14-6 to get OH concentration concept: The [H+] and [OH−] concentrations are related by the self-ionization constant Kw of water according to the expression Kw = [H+][OH−] = 1.0 × 10−14 M2. Using a logarithmic approach, pH is related to [OH−] by the relationship pH + pOH = 14, where pOH = −log[OH−].

q37

1 atm = 760 mmHg = 760 torr = 101,325 Pa = 101.325 kPa concept: Pressure measurements may be stated in a variety of units, including pascals (Pa), kilopascals (kPa), atmospheres (atm), millimeters of mercury (mmHg), and torr. These units can be converted from one to another by using appropriate conversion factors, given that 1 atm = 760 mmHg = 760 torr = 101,325 Pa = 101.325 kPa.

question 27

1. Look at the atom. 2. Count the number of atoms connected to it (atoms - not bonds!) 3. Count the number of lone pairs attached to it. Lone pair are treated as 1 electron not 2 electrons. And make sure to count the hydrogen in carbons where its not shown (if it exists bc carbon doesn't have lone pairs). Also, double and triple bonds are counted as 1 atom not 2 or three. 4. Add these two numbers together. If it's 4, your atom is sp3. If it's 3, your atom is sp2. If it's 2, your atom is sp. (If it's 1, it's probably hydrogen!) concept: Hybrid orbitals are formed by combining the atomic orbitals of an atom. The hybridization of an atom is determined by counting the electron domains (sigma bonds and lone pairs) and assigning a hybrid name. The sum of superscripts must equal the number of electron domains.

q40

A phase diagram shows a substance's stability in each phase as a function of pressure (y-axis) and temperature (x-axis). The boundary lines between two phases indicate the conditions under which they are in equilibrium. According to the passage, Raoult law states that the addition of any solute to a pure liquid lowers the freezing point and vapor pressure of the resulting solution. A substance's phase diagram shows this change by moving the phase boundaries to lower temperatures and pressures, respectively. concept: A phase diagram consists of phase boundary lines that illustrate the equilibrium points and transitions between phases as a function of pressure (y-axis) and temperature (x-axis). Raoult law states that the addition of solute to a pure substance lowers the freezing point and saturation vapor pressure of the resulting solution. On addition of a solute, the phase boundary lines on a phase diagram should move accordingly.

Q5

Accordingly, consider the equilibrium: H2PO^4− ⇄ HPO2^−4 + H+ At a pH of 8.0, HPO4^2− will be the dominant species in the phosphate buffer because the pKa of H2PO^4− is 6.8, which is less than 8 so the species would be DEPROTONATED. Likewise, consider the equilibrium: NH4+ ⇄ NH^3 + H+ In this case, the acidic NH4+ will be the dominant species in the ammonia buffer because the pKa of NH4+ is 9.1, which is greater than 8 so it would be PROTONATED

question 24

Accordingly, lower pOH values indicate more alkaline solutions with higher [OH−]. This is opposite to the pH scale, in which lower pH values indicate more acidic solutions. Consequently, pH and pOH are inversely correlated. As pH rises, pOH falls (and vice versa) while maintaining the relationship pH + pOH = 14. Going from the highly acidic stomach to the less acidic duodenum gives a higher pH and a lower pOH, meaning [H+] decreases and [OH−]increases. Concept: Analogous to the pH scale, the pOH scale is defined as pOH = −log[OH−]. The pOH scale assesses acid-base measurements with respect to [OH−] whereas the pH scale operates with respect to [H+]. The pH and pOH scales are inversely correlated (as one decreases, the other increases), such that pH + pOH = 14

q7

As stated in the passage, organic sunscreens are aromatic compounds that protect skin by absorbing ultraviolet (UV) radiation. The electrons that absorb energy during this process enter electronically excited states and must subsequently relax back down to the ground state. This occurs by the emission of the excitation energy as heat. concept: Electrons occupy orbitals around a nucleus, each with a specific energy. Electrons can jump between energy levels by absorbing or emitting energy exactly equal to the difference in energy levels. After excitation, electrons always relax back to the ground state (ie, lowest allowed energy level).

Q9

At saturation, the NaC9H7O4 solution contains 120 g of NaC9H7O4 in every 100 mL of water. This ratio equates to a concentration of 1.20 g/mL. To convert this into molarity, the volume must be converted from milliliters to liters (1000 mL = 1 L), and the mass (grams) must be converted into moles using the molar mass of NaC9H7O4 (202.2 g/mol). Multiplying 1.2 g/mL by the volume conversion and dividing the grams by the molar mass yields a saturating molarity of 5.9 mol/L. concept: A saturated solution contains the maximum amount of compound that will dissolve in a given volume of solvent under the given conditions. Solubility (expressed as grams of solute per 100 g of solvent) can be stated in units of molarity (moles of solute per liter of solvent) using unit conversions and the molar mass of the solute.

q18

Atoms that pack together more tightly achieve a greater density than molecules with a less space-efficient packing arrangement. Accordingly, the density of an element can be calculated based on its atomic radius and the atomic packing within its unit cell. Q is asking for represetation of denisty so d=m/v they give you mass and you had to know that Volume of the cube (unit cell) V=S^3 = (2r)^3 cuz 2r = 1 side diameter of the cube concept: The density of a substance varies slightly with temperature and is defined as the mass of the substance per unit of volume that it occupies. The density of an element can be calculated based on its atomic radius and the atomic packing within its unit cell.

q44

Based on the passage, the nitration of tyrosine in a lipid environment is a two-step reaction. Looking at the lipid environment delta G on the table, the first step has a ΔG of −91.8 kJ/mol with a large activation energy, and the second step has a ΔG of −14.1 kJ/mol with a smaller activation energy. So, the first step will show a higher delta G than the second step which is smaller and this is choice D concept: The reaction coordinate diagram is a representation of the change in energy of a system during a chemical reaction. The relative energies of the reactants, transition states, intermediates, and products are illustrated as their respective heights on the diagram.

question 25

Because the pH scale is logarithmic, the magnitude of [H3O+] changes by a power of 10 for each pH unit. The factor by which [H3O+] changes has an exponential relationship to the change in pH (the difference between the final pH and initial pH) as given by: Factor Δ[H3O+] = 10^−(final pH − initial pH) Moving from the parietal cell (pH 7.0) to the stomach lumen (pH 1.0) involves a change of 6 pH units. The increase in [H3O+] going from the parietal cell to the stomach lumen can be determined as follows: Factor Δ[H3O+] = 10^−(1 − 7) = 10^6 concept: The pH unit is based on a logarithmic scale defined as pH = −log[H3O+]. The magnitude of [H3O+] changes by a power of 10 for each pH unit and is given by the expression Factor Δ[H3O+] = 10−ΔpH.

question 16

Bond dissociation energy is the energy required to break a bond. It tends to increase with each additional pi bond. Single bond = lowest bond dissociation energy Double bond = medium Triple = highest bond dissociation energy Shorter bonds are stronger and require more energy to break. Atoms with small atomic radii can form short, strong bonds whereas atoms with larger radii form longer, weaker bonds. concept: The energy required to break a bond, known as the bond dissociation energy, is related to the bond length. Atoms with smaller radii (near the top of the periodic table) tend to form shorter, stronger bonds than atoms with larger radii.

q10

Careful analysis of data and what it implies is important to form conclusions from experimental results. A correct understanding of what the data indicate is essential for drawing correct conclusions.

q4

Compound 1 has two N-H bonds capable of hydrogen bonding, whereas Compound 2 has only one N-H bond. Having fewer N-H bonds decreases the extent of hydrogen bonding and lowers the energy required to overcome the intermolecular attractions. As a result, the boiling point of Compound 2 is lower than that of Compound 1 making A the only answer. Concept: Structures with a lesser extent of intermolecular forces have lower boiling points because less energy is required to overcome the attractions between molecules. When comparing similar structures, the boiling point tends to decrease as the number of hydrogen-bonding groups decreases due to fewer hydrogen bonding interactions.

Q30

Concept: Parts per million (ppm) is a dimensionless quantity used to express concentrations as a fraction out of a basis of 1,000,000. In aqueous solutions, 1 ppm = 1 mg/L, and for concentrations expressed as a percentage, 1% = 10,000 ppm.

question 8

Concept: Solution concentrations can be expressed as the mass of a given substance per unit of solution volume. The mass of a substance contained in an aliquot can be calculated by multiplying the aliquot volume by the solution concentration when both values are expressed with respect to the same units of volume.

Q29

Concept: Solvent diffusion across the semipermeable membrane during osmosis produces an osmotic pressure against the membrane proportional to the osmolarity (concentration of osmotically active species that cannot cross the membrane) rather than to the solute molarity due to the dissociation of some solutes. Solute concentration and osmolarity are related by the van 't Hoff factor.

Q28

Concept: The diffusion of solvent across a semipermeable membrane during osmosis produces an osmotic pressure against the membrane, which is directly proportional to the concentration of the osmotically active solute in solution.

q33

Could use collision theory to think abt this problem. Collision theory says that molecules must collide to react. The collisions must have sufficient kinetic energy to overcome the activation energy (Ea) of the rxn. This concept is described mathematically using the Arrhenius equation In the sodium iodide-catalyzed decomposition of H2O2, I− ions must collide with H2O2 for the catalyzed reaction to occur. Changing the catalyst's cation from Na+ to K+ does not change the concentration of I− or the concentration of H2O2, and therefore will not change the frequency of collisions in the reaction. Furthermore, the data in Figure 2 will remain the same because changing the catalyst cation will not affect the heat evolved from the reaction. Educational objective: The collision theory of kinetics assumes that molecules must collide for a reaction to occur. The collisions must have enough kinetic energy to overcome the activation energy. The reaction rate can be described using the Arrhenius equation

q27

Due to differences in orbitals and the number of valence electrons, H2, F2, Cl2, Br2, and I2 can achieve a full valence shell by forming a pair with only a single covalent bond, but O2 must form a double bond and N2 must form a triple bond. In the second period (row) on the periodic table, nitrogen, oxygen, fluorine, and neon are all elemental gases, but only oxygen exists as a diatomic molecule with a double covalent bond in its elemental state. concept: Of the common representative elements, hydrogen, nitrogen, oxygen, fluorine, chlorine, bromine, and iodine occur naturally as covalently bonded diatomic molecules in an elemental state to achieve a full valence shell. Each of these elements exists as a gas at room temperature except for bromine (a liquid) and iodine (a solid).

q13

During chemical reactions, molecules convert their internal (chemical) energy into thermal energy (heat) as new bonds form, and convert thermal energy into internal energy as old bonds break. More heat released than consumed = exothermic = negative enthalpy More heat consumed than released = endothermic = positive enthalpy. The value given in the psg for enthalpy is negative meaning is heat is being released. The temperature/ time graph is showing that heat and since its increasing you would get a positive slope Concept: The change in enthalpy of a reaction ΔH° determines whether heat is released or consumed. A negative value for ΔH° corresponds to an exothermic reaction in which heat is released and temperature rises. A positive value indicates an endothermic reaction and a decrease in temperature.

Q4

During the gamma decay of an atom of technetium-99m, the nucleus emits a high-energy photon (Number I), but the number of protons and neutrons in the nucleus remains unchanged (Number II). III is wrong bc the number of protons in the nucleus is unchanged, the atom will not be converted into an isotope of a different element. concept: Radioactivity is the spontaneous emission of radiation (photons or particles) from an unstable atomic nucleus via common decay processes such as alpha decay, beta decay, and gamma emission. During gamma emission, a high-energy photon is released from a nucleus in an excited state without the release of a particle, which causes the elemental identity of the nucleus to remain unchanged.

Q17

Electrodeposition equation (on the equation sheet) --> mol M = It/nF concept: In an electrolytic cell, the deposition of solids as a result of the reduction of a metal ion onto the cathode is known as electroplating. The number of moles of metal plated can be found for a given current over a given amount of time using the equation that relates current, moles of electrons, and time: mol metal=It/nF

q9

Fastest way to do this is by checking the first decay and if its not right using POE and moving on to the next choices. concept: In alpha decay, the mass number of an atomic nucleus decreases by 4 units and the atomic number decreases by 2 units. In all forms of beta decay, the mass number of an atomic nucleus remains unchanged, but the atomic number increases (β− decay) or decreases (β+ decay) by 1 unit.

q45

For elementary reactions (one-step reactions, individual steps of a multistep reaction), the rate law is written as the product of the rate constant k and the concentration of the reactants, in which the stoichiometric coefficient of each reactant is its exponent. If the reaction is not elementary (ie, requiring more than one step), the slowest elementary step (or rate-determining step) determines the rate law for the entire reaction. This step can only be determined experimentally. If the initial step is the slowest, the rate law is determined simply from that step using the coefficients of each reactant as exponents in the rate law. However, if the slowest step is the second step, reaction intermediates play a role in the rate law. The nitration of tyrosine is a multistep reaction in which the first step is the rate-determining step. Therefore, the rate law can be written using the reactants of step 1: Rate = k1[tyrosine][O3] concept: Rate laws for molecular reactions relate the rate of reaction to the concentrations of reactants. For a multistep reaction, the rate law must be determined experimentally by determining the rate-limiting step. Multistep reactions are determined according to the slowest or rate-determining step.

q41

Freezing occurs when the kinetic energy of a molecule can no longer overcome the intermolecular forces binding it to nearby molecules. If a molecule experiences strong intermolecular forces with surrounding molecules, the kinetic energy required to overcome those forces is greater, and the freezing point temperature is correspondingly higher. Therefore, the freezing point of water is impacted by the strength of intermolecular forces Raoult law states that for an ideal solution, the addition of a solute lowers the freezing point of a liquid according to the amount of solute added. concept: The freezing point of a substance is determined by the strength and extent of intermolecular forces acting between molecules. These interactions are affected by the presence of solutes. As a colligative property, the extent of freezing point depression is a function of the amount of solute but not the solute's chemical properties.

q45

From passage had to look for molecules Educational objective: The number of moles of a compound is found by dividing the mass of the sample by its molar mass. Stoichiometric mole ratios taken from a balanced reaction equation or a chemical structure can be applied to determine the number of moles of one chemical species relative to the moles of another chemical species in the reaction.

q1

Have to know the oxidation rules and do it out concept: Using oxidation state rules, the oxidation number of each atom in a compound formula may be determined on the basis that the sum of the oxidation states of each atom must equal the net charge of the compound or ion.

q51

Have to remember: Higher the ionization energy = least reactive Trend for ionization energy : increases as you go up and left to right Since Li has the highest 1st ionization energy in group 1, it will be the least reactive alkali metal concept: The first ionization energy is the energy required to remove the first, most loosely bound valence electron from a neutral atom. The first ionization energy tends to increase with increasing atomic number moving across a period and decrease moving down a group on the periodic table. Elements with a lower first ionization energy are easier to ionize in reactions than elements with a higher ionization energy.

question 21

Hydrophobic character increases as the number of non-polar molecule increases and the number of methyl groups or carbons increases. As the number of polar groups increase, the molecule gets more hydrophillic. Thats why the answer is D bc it has 4 methyl groups. concept: Molecules with many polar bonds that promote dipolar interactions with water are hydrophilic, whereas those with mostly nonpolar bonds that lack attractive dipolar interactions with water are hydrophobic. The overall character of a compound depends on the relative number of each type of bond present.

q2

I got the question wrong bc i went too fast and put Mn as -6 instead of +6. When you do it out you get O2 as the oxidizing agent bc it goes from 0 charge to -2 charge. When doing these problems compare the elemental charge not the total charge Oxidizing agents are reduced in the process (they gain electrons = becomes more negative) Reducing agents are oxidized in the process (they lose electrons = becomes more positive) Concept: By comparing the oxidation number for a given element in the reactants with the oxidation number of that same element in the products, oxidation and reduction processes can be identified. Oxidizing agents cause oxidation in other atoms while the agent itself gets reduced in the process.

Q10

If 0.8 g need to be delivered into the reaction, the number of moles in this amount of mass can be found by dividing by the molar mass of CuSO4: (0.8 g) / (159.6 g/mol) = 0.005 mol CuSO4 To find the volume of a 0.50 M solution required to obtain 0.005 mol CuSO4 for delivery into the reaction, divide the number of moles by the molarity of the solution: (0.005 mol CuSO4) / (0.50 mol/L) = 0.01 L CuSO4 solution

q43

If 18-O absorbed a proton, the atomic number would increase by 1 to a total of 9 and all atoms with 9 protons are fluorine atoms the resulting nucleus would be a flourine isotope. Isotopes differ in the number of neutrons therefore they differ in physical properties but not in chemical properties Educational objective: Isotopes are atoms of the same element that have the same number of protons but a different number of neutrons in the nucleus. Isotopes of the same element have nearly identical chemical properties but differ in their physical properties.

q42

If 18F went through a positron emission (b+ decay) the atomic number would decrease by 1. So F would become O and thats choice A showing the connectivity of OH where F would normally go on the structure concept: Positron emission occurs during β+-decay, in which a nuclear proton converts to a neutron. This causes the atomic number of the nucleus to decrease by 1 while the mass number of the nucleus remains unchanged.

q23

If radium-226 underwent an electron capture, the result would yield a nucleus of the same mass number (226) but an atomic number that is 1 less (88 − 1 = 87). Therefore, francium-226 would be detected. concept: In all forms of beta decay, the mass number remains unchanged, while the atomic number increases (β−-decay) or decreases (β+-decay and electron capture) by 1. β−-decay converts a neutron into a proton and emits an electron. β+-decay and electron capture convert a proton into a neutron (the opposite of a β−decay).

Q13

If the concentrations of [X] and [Y] are such that [X]c[Y]d < Ksp, the solution has not reached the solubility limit and will permit more ions into solution (dissolve more of Z). Conversely, if the concentrations of [X] and [Y] are such that [X]c[Y]d > Ksp, the solubility limit of the solution has been exceeded and the solution will not accept any more ions (no more Z will dissolve). This will cause any ions in excess of the Ksp to come out of solution as a precipitate. concept: In a solution formed from the dissolution of a compound into ions, the compound will dissolve until the solution reaches the value of the solubility product constant Ksp for the compound. Ion concentrations that exceed the Ksp result in precipitation of the excess.

q55

In a reaction free-energy diagram, transition states are seen graphically as energy maxima (peaks). The highest energy transition state (tallest peak relative to the reactant energy) is the greatest energy barrier for the overall reaction and determines the activation energy Ea. Therefore, Ea is the energy difference between the highest peak and the reactants (peak energy minus reactant energy). Educational objective: The activation energy is the minimum energy needed to reach the highest energy transition state that enables the completion of a reaction. In a reaction free-energy diagram, the activation energy is the energy difference between the tallest peak (relative to the reactants) and the reactants (peak energy minus reactant energy)

q30

In an oxidation-reduction (redox) reaction, the oxidation states of some atoms change during the conversion of the reactants to the products. Just had to notice that it goes from a solid nickle metal so (0) for ox# to nickel (II) bromide to Ni2+ so oxidation rxn Educational objective: Many reactions can be classified as one of five common types, but they can also be further classified based on the results of the reaction. Classifications include precipitate formation, chelate formation, and oxidation-reduction reactions.

question 1

In the passage we were given the concentration of oxytocin monoacetate - .3mg/mL To find the volume you have to do molar conversion concept: Solution concentrations are given in terms of the amount of solute per unit volume of solution. Solute amounts are typically given in terms of moles or mass, whereas solvent amounts are given in terms of volume. The volume of a solution containing a certain amount of solute is found by converting the solute amount to matching units of mass (or moles) and then dividing by the solution concentration

q8

In the psg it says that platinum increases the rxn rate. Which means that its working as a catalyst by stabilizing the transition state. concept: The transition state of a reaction is unstable and presents an energy barrier to the reaction's progress. A catalyst stabilizes the transition state, and therefore increases the reaction rate without being consumed.

q36

In this passage, Frost diagrams are presented as a way of predicting which chemical species are likely to undergo disproportionation. As shown in Figure 2 of the passage, a species is likely to undergo disproportionation if its position on the Frost diagram lies above a line connecting the points of two adjacent species because ΔG° < 0 with respect to the mean of the potential gradient. Accordingly, answering this question is a matter of graphical evaluation. Based on the above criteria and an examination of Figure 1, MnO2 is least likely to undergo disproportionation because its point on the Frost diagram lies below the line connecting the points of two neighboring species. Educational objective: A disproportionation reaction is a redox reaction in which both the oxidation and the reduction occur with atoms of the same element. The tendency of a reaction toward disproportionation can be evaluated graphically.

q8

Instead of doing out the percentages, use some logic and save yourself time: For the unidentified element in this question, the majority of the sample is isotope A, which has a mass number of 20. Therefore, the atomic weight (relative atomic mass) measured for the sample will be closest to 20 amu because isotope A is predominant within the sample. concept: In a naturally occurring sample of an element, a mixture of isotopes is present in percentages resulting from their respective natural abundance. The atomic weights (relative atomic masses) listed on the periodic table account for natural isotope abundance by taking a weighted average of the mass of each isotope naturally found in a sample.

question 41

Just had to balance the equation concept: The conservation of mass dictates that the number of any type of atom be the same on both sides of a chemical equation (the equation must be balanced).

q10

Just had to check alpha and beta decay concept: In gamma decay, a high-energy photon is released from a nucleus and the mass number, atomic number, and elemental identity remain unchanged. In all forms of beta decay, the mass number of an atomic nucleus remains unchanged but the atomic number increases (β− decay) or decreases (β+ decay, electron capture) by 1 unit.

question 46

Lewis Acid means it accepts electrons so the charge on the molecule has to be positive. And bronsted-lowry acid donates H+ but they are saying NOT a bronsted-lowry acid so that means you are looking for something that wouldn't have a H attached to it bc its not donating the H+. Concept: A Lewis acid is a molecule that acts as an electron pair acceptor, and a Lewis base is a molecule that acts as an electron pair donor. The Lewis definitions more broadly define acids and bases to include some compounds that lack acidic protons.

q21

Look at PT and be able to write the electron config from Ne onwards and since Ca2+ with 2+ charge, it will have the same electron config of Ar which is: [Ne]3s^2 3p^6 Educational objective: An electron configuration accounts for all electrons held by an atom or ion, listed sequentially by shell and subshell in order of increasing energy. A noble gas abbreviation may be used to represent the configuration of lower-level electrons. Ionized atoms lose electrons from higher energy levels first."

q56

Looked at fig 2 and knew that the largest energy barrier would be the RLS and thats peak 1 which is teh first step. The free energy diagram (Figure 2) of the reaction in Figure 1 shows that forming either product involves two steps. The energy barrier for the conversion of Compound 1 into the intermediate (Step 1) is much larger than either of the energy barriers to convert the intermediate into either product (Step 2). Therefore, the conversion of Compound 1 into the intermediate (Step 1) is the rate-determining step (slowest step with the smallest value of k) and determines the activation energy for the reaction. Educational objective: The reaction rate is proportional to the rate coefficient k, which decreases exponentially as the energy needed to reach the transition state increases. In a reaction involving two or more steps, the step that must overcome the largest energy barrier to pass through the transition state is the rate-determining step.

question 52

My answer is wrong bc tyrosine is amphipatic so it can be hydrophillic and thats a big change in water solubility. Whereas isoleu to valine are the same type and that wont cause a big change in water solubility concept: Water solubility requires that a compound has enough polar intermolecular interactions between the structure of a compound and the water molecules that the compound can be solvated and carried into solution. Hydrophobic molecules tend to be predominantly nonpolar and are sparingly soluble in water. Structural changes that increase polar sites on a molecule will enhance water solubility.

Differences btw diamagnetic and paramagnetic

Paramagnetic = unpaired electrons Diamagnetic = no unpaired electrons

q47

Passage based Q so P4 states that limitations in the spatial resolution of PET scanners can cause objects (eg, tumors) with a diameter less than 30 mm to appear smaller than the actual size because the PET scan does not detect all of the 18F-FDG present in the localized tissue. They quantify this by using the recovery coefficient given in the same passage RC=(SUVmeasured/SUVactual)x100 Look at fig 3 for 1.7cm tumor given in qstem so thats 17mm and can find that its RC in percent is 80% and you have actual SUV from qstem so you solve for SUV measured Educational objective: Data plots that correlate different measurements can be used to estimate the expected result for similar measurements under the same conditions.

q16

Psg gives you ΔH° (−55 kJ/mol) and ΔS° (−0.15 kJ/mol·K) of the overall reaction. Have to knwo the Gibbs free energy equation ΔG° = ΔH° − TΔS° and the temperature at which ΔG° = 0 can be calculated by setting up the equation: 0 kJ/mol = −55 kJ/mol − T(−0.15 kJ/mol·K) 55 kJ/mol = −T(−0.15 kJ/mol·K) look at math --> The initial answer will be in kelvin. To convert the temperature to Celsius, subtract 273 from the answer in kelvin: 367 − 273 = 94°C ≈ 95°C Educational objective: The Gibbs free energy of a reaction determines whether the reaction is spontaneous. It is related to both enthalpy and entropy by the equation ΔG° = ΔH° − TΔS°. A negative ΔG° corresponds to a spontaneous reaction whereas a positive ΔG° corresponds to a nonspontaneous reaction.

q11

Psg stated that platinum increases the rxn rate which means that it acts as a catalyst so making it into a finer powder increases the SA of teh catalyst and increases the speed of the rxn and the resulting power output. Educational objective: Catalysts provide a surface on which stable transition states can form. A greater catalytic surface area corresponds to a greater reaction rate.

Q21

Pure liquids and solids are never included in these equations. Thats why my answer was wrong concept: Acid ionization in an aqueous solution produces hydronium ions and a corresponding conjugate base of the ionized acid. Both the acid and its conjugate base interact with water and establish equilibria that can be described by molar concentration ratios in the form of the acid dissociation constant Ka and the base dissociation constant Kb, respectively.

Q16

RED CAT AN OX OIL RIG - Oxidation = loss of electrons; Reduction = gain of electrons Electrons flow from anode to cathode PbSO4 is the solid product that accumulates on both electrodes after the battery has been discharged. When the lead-acid battery is charging, electrons flow toward the lead electrode, making it the cathode. Because reduction occurs at the cathode, PbSO4 is reduced. Concept: When a battery is discharging, it behaves as a galvanic cell, and when it is charging, it behaves as an electrolytic cell. In either case, oxidation occurs at the anode and reduction occurs at the cathode.

question 6

Rules for determining hybridization: 1. Look at the atom. 2. Count the number of atoms connected to it (atoms - not bonds!) 3. Count the number of lone pairs (not individual electrons) attached to it. 4. Add these two numbers together. If it's 4, your atom is sp3. If it's 3, your atom is sp2. If it's 2, your atom is sp. (If it's 1, it's probably hydrogen!) concept: During hybridization, atomic orbitals combine to form new hybrid orbitals that assume spatial orientations to minimize the electronic repulsion between orbitals. The number of hybrid orbitals that form and the number of orientational vectors adopted by the hybrid orbitals will equal the number of atomic orbitals that combine.

q39

Several important characteristics contribute to water acting as a solvent: 1) Because oxygen is more electronegative than hydrogen, the bonding electron pairs are more tightly drawn to oxygen. This property results in the oxygen atom having a partial negative charge whereas the hydrogen atoms have a partial positive charge, allowing interactions with other charged molecules 2) The bent geometry of water contributes to its polarity by grouping positive charges at one end of the molecule and negative charges at the other. The charged regions of a water molecule are attracted to opposite charges on other polar compounds 3) Water's small size allows efficient formation of a hydration shell around solutes. This process acts to evenly distribute and isolate solute particles concept: As a polar compound, water is an efficient solvent due to its ability to interact with other polar or charged compounds. The polar nature of water is due to the difference in the electronegativities of oxygen and hydrogen, along with a bent geometry. The small size of the water molecule allows efficient interactions with solutes, forming a hydration shell around and dissolving them.

q5

Since they are testing the activity for Gd-daa3 in the presence of the cations, to get the baseline activity (control) you want to measure the activity without the cations present concept: The experimental control is a measurement taken with all experimental conditions in place except the independent variable of interest. Controls provide baseline measurements so that the effects of independent variables on dependent variables can be accurately determined.

q 2

Since they asking for R, I just used PV=nRT and the solved for R which gave PV/RT and then i just used the units from the graph to find the answer You can also look at the slope (rise/run) and then match it up with the units concept: The average kinetic energy of a single ideal gas molecule is directly proportional to temperature via the Boltzmann constant, which can be expressed per mole of gas as the ideal gas constant R. According to the ideal gas law, PV is directly proportional to nT, and a graph of PV vs. nT results in a straight line with a slope of R.

q59

Small atoms such as oxygen and fluorine have several electrons crowded around a small nucleus, resulting in greater electron-electron repulsion. Adding another electron would increase repulsion forces even further, so it is more difficult to add an electron to oxygen than to a larger atom with lower repulsion forces. concept: Electron affinity is the measure of energy change when an electron is added to an atom in the gaseous state. Electron-electron repulsions result in a decreased electron affinity.

q54

Step 1 transition state is the first transition state which is D concept: A reaction free-energy diagram is a plot of the Gibbs free energy (y-axis) versus the reaction coordinate (x-axis). During elementary molecular conversion steps, existing bonds are broken while new bonds are formed, resulting in momentary transition states with elevated energy that are seen graphically as energy maxima (peaks).

q49

Temperature change means that you are changing the equilibrium constant (Keq). The effect of temperature on an equilibrium reaction can be illustrated by treating heat as a chemical reagent and applying Le Châtelier principle. Step 1 of the nitration of tyrosine in an aqueous environment is an exothermic reaction, and heat can be viewed as a product of the reaction. Raising the temperature causes a shift toward the reactants as heat is consumed to rebalance the equilibrium reaction. The shift toward reactants leads to an increase in reactant concentration, and therefore a decrease in Keq. concept: Equilibrium is reached in a reaction when the rate at which products are formed is equal to the rate at which reactants are formed. The equilibrium constant Keq is defined as the ratio of the reactants and products when a reaction is in equilibrium, and only changes in temperature can alter Keq. Le Châtelier principle states that a system responds to disturbances in equilibrium by shifting the reaction to counter the effect of the disturbance.

q23

The fastest way to do this would be to make an educated guess bc the other numbers are too high. Proper way to do it: Find half of 3112 and match with 1556 at 15 hrs on the table. 15 - 3 = 12 so 12 is the half life. since at 12 hours the activity is 1850, you just multiple 1850x2 to get the answer at 0 hours Educational objective: The activity of a radioisotope (the number of decay events per unit of time) is proportional to the amount of the decaying radioisotope present and decreases by 50% for each radioisotope half-life that passes. With each subsequent half-life, the initial amount of isotope that remains becomes exponentially smaller and eventually approaches zero."

q29

The graph is showing a spontaneous reaction bc the energy of the reactants is more than the energy of the products. For a spontaneous reaction, the enthalpy of the reaction is negative and the activation energy is positive bc it is the difference btw the transition state and the energy of the reactants. concept: The difference in energy between the products and the reactants is the heat of the reaction (enthalpy). The enthalpy is negative for exothermic reactions and positive for endothermic reactions. The difference in energy between the transition state and the reactants is the activation energy, which is always a positive value.

What is the equation to find the effective nuclear charge (Zeff)?

The nuclear charge Z influencing a valence electron is diminished by the shielding constant S, which results in an effective nuclear charge Zeff that is less than Z and increases as the atomic number increases. By assuming S is equal to the number of core electrons, Zeff can be estimated as: Zeff = Z - S

Question 14

The passage states copper ions in solution express an affinity to form coordinate bonds with available lone-pair electrons of nitrogen atoms in amines (Reaction 2). If excess amine ligands are present with Cu2+(aq) ions, each Cu2+ ion in solution will form coordinate bonds with four nitrogen atoms. This will disrupt the equilibrium. The decrease in Cu2+(aq) will cause the equilibrium to shift toward the products to form more Cu2+(aq). Consequently, on adding excess methylamine, the amount of CuF2(s) that dissolves will increase. Concept: Adding ligands that can form soluble coordination complexes with metal ions in a solution can enhance the solubility of ionic salts. This occurs by applying Le Châtelier's principle to disrupt the solubility equilibrium and shift the reaction toward the products.

Q18

The passage states that a NiCd battery produces a potential of 1.3 V, so the reverse reaction would require more than 1.3 V of applied potential to drive the reaction in the non-spontaneous direction. concept: When charging a battery, an external potential must be applied to force the oxidation-reduction reaction in the nonspontaneous direction. The reverse reaction requires more than the potential produced by the battery because of internal resistance.

q31

The periodic table shows that strontium has an atomic number of 38 whereas yttrium has an atomic number of 39. Because the atomic number increases by one, strontium must gain a proton by emitting a nuclear electron. concept: The atomic number of a nucleus but not its atomic mass can be changed by the emission of a nuclear electron or positron, or by electron capture. Electron emission increases the atomic number by one whereas positron emission and electron capture both decrease the atomic number by one.

q39

The question is asking for the formal charge of chlorine and oxidation state of oxygen Draw out the lewis structure (should - has) And follow the oxidation rules

Q8

The question is asking what type of solution will CH3COONa form when it breaks apart or goes through hydrolysis. CH3COONa is a basic salt produced by a weak acid CH3COOH and a strong base NaOH. So, the hydrolysis of the salt will produce a basic solution with the pH > 7 Concept: The pH of a salt solution is determined by the extent of hydrolysis caused by the nature of the salt. Neutral salts yield solutions with pH = 7, acidic salts give solutions with pH < 7, and basic salts produce solutions with pH > 7.

question 42

The theoretical yield of a chemical reaction is the maximum amount of product possible based on stoichiometric calculations. The theoretical yield is based on the number of moles of the limiting reactant and the stoichiometric ratios in the balanced chemical reaction. The actual yield obtained after conducting a reaction will be less than the theoretical yield when some of the product is lost in the process of conducting laboratory procedures such as filtering or transferring. The actual yield could appear to be greater than the theoretical yield when a product contains impurities or has not been completely dried before weighing. In the reaction between NaCl (table salt) and AgNO3, solid AgCl is produced. If some of the solid AgCl is lost during filtration to isolate the product, the actual mass of the product obtained will be less than the calculated theoretical mass. According to the equation: (actual yield/theoretical yield) ×100= % yield If the actual yield is less than the theoretical yield, then the percent yield will be less than 100% Concept: The percent yield is a description of the efficiency of a reaction. It is the ratio of the mass obtained from an experiment (the actual yield) to the calculated mass (theoretical yield) expressed as a percentage.

q15

They are asking for the mass og calcium in sample 1. Table 1 shows the absorbance of sample 1. You had to match that absorbance in the graph to find the concentration of Ca2+ in M. After finding the concentration you had to convert it into mL and use the total mL of solution given in the passage (50mL). That gives you the mol of ca2+ and then you use the molar mass to find the grams of Ca2+ concept: The analyte concentration in an unknown sample can be determined by comparing its absorbance to a calibration curve prepared using the measured absorbances of standard solutions with known concentrations. The determined concentration may then be used to calculate the mass of the analyte present in the sample based on the solution volume, relevant mole ratios, and the molar mass of the analyte.

q22

To calculate the Zeff experienced by the valence electrons of a nitrogen atom with 7 protons (Z = 7) and 2 core electrons (S = 2) gives: Zeff,N = Z−S = (7−2) = +5 To find the number of core electrons you do: total number of electrons (the atomic number) - valence electrons

question 8

To draw out the structure, start with the carbons and then add your hydrogens and oxygens. To find the formal charge = should - has. concept: A formal charge is assigned to an atom based on an electron accounting method that assumes the bonding electrons between two atoms are shared equally. Formal charge equals the group valence of an atom minus both the number of nonbonding electrons and half of the number of bonding electrons.

q16

To find if sample 2 is considered hard had to look at table 1 for the absorbance to match up that absorbance with the concentration in the graphs for Ca2+ and Mg2+. Using the absorbances in Table 1 and the calibration curves in Figures 1 and 2, the concentration of Ca2+ in Sample 2 is 0.003 M and the concentration of Mg2+ is 0.0003 M. Water hardness is determined using Equation 1 given in the passage: [CO32−] = 2.5[Ca2+] + 4.1[Mg2+] Plugging the concentrations into the equation gives: [CO32−] = 2.5 (0.003 M) + 4.1 (0.0003 M) [CO32−] = 0.0087 M Since .0087 M > .002 M = sample 2 is considered very hard water concept: The concentration of an analyte in an unknown sample can be determined by comparing it to a calibration curve prepared from samples of known analyte concentration. The determined concentration may be used in empirical relationships to assess sample characteristics.

q35

We are starting with an initial amount of 20ng and they are asking how long will it take for it to decay to 2.5 ng. Start with 20 and keep diving the number by 2 to get to 2.5 20 --> 10 --> 5 --> 2.5 3 half lives passed and the half life of 90Y is 64 years 64 x 3 = 192 hours concept: For radioactive particles, the half-life t1/2 is the amount of time needed for half the quantity of a substance to decay. The fraction remaining after a number of half-lives can be calculated using a formula for exponential decay, 1/2^n, where n is the number of elapsed half-lives.

q50

We are talking about hydrogen here and they are asking which one would be most similar to hydrogen. The answer is lithium bc its in the same group and so they would have similar electron configurations and similar chemistry concept: The number of valence electrons drives chemical behavior. Atoms tend to gain, lose, or share electrons to achieve a full valence shell. Elements within the same group of the periodic table have the same number of valence electrons and exhibit similar chemical properties.

Q19

When a lead storage battery is discharged, lead oxide (PbO2) gains electrons, or is reduced. Based on the oxidation-reduction reactions for the lead storage battery, water is one of the products from the reduction of PbO2. This oxidation-reduction reaction occurs in an acidic medium. Before discharging, the concentration of H2SO4 is 4 M, but after the battery is completely discharged, the acid is diluted by the additional water, lowering the concentration of H2SO4. Because the concentration of H2SO4 changes when the battery is discharged, it is often used as an indicator of when the battery needs to be charged. Concept: A lead storage battery is an example of an oxidation-reduction reaction that occurs in an acidic electrolyte. Because water is one of the products of the reaction, the acidic electrolyte will be diluted as the battery discharges.

q34

You gotta do the reaction quotient (Q) for the equation given (same way you would set up keq for an equation). And then you can see that O2 is raised to the 4th power bc it has a coefficient of 4 in the equation. So assuming that [O2] is initially x mol/L and is then doubled to 2x mol/L, the change will be raised to the power of 4 ([2x]^4 =16x^4). Because [O2] is in the denominator of the Q expression, this will decrease the value of the reaction quotient by a factor of 16 relative to the initial value of Q. concept: The law of mass action states that a reaction quotient Q can be expressed as a ratio of the molar concentrations of the products over the reactants, each raised to the power of its respective balanced reaction coefficient. Reactions tend to proceed toward equilibrium by shifting species concentrations until Q equals the equilibrium constant Keq.

question 38

You had to read the paragraph to see what produced what. In this question, calcium carbonate is isolated from the inactive ingredients in an antacid tablet and then placed over a flame. The gas produced is CO2, based on the results from the glowing splint test. This means that the white solid that remains is a new compound that does not have carbon in it. Calcium preferentially forms a metal oxide in the presence of oxygen, so the other product must be CaO. Concept: A decomposition reaction is a type of reaction in which energy is added to break down a compound into either its elemental constituents or into simpler, stable compounds.

q38

You just need to look at the answer choices and match it up with the phase diagram and the table. Liquid water initially at 274 K and 6.5 mb will cross the liquid-gas boundary line when heated to 313 K (the temperature at the Spirit location according to the table), so liquid water will boil under these conditions. A: Conditions at the Viking I location are 256 K and 7.11 mb. Under these conditions, water enters the solid phase. B: Sublimation can occur only at pressures below the triple point. Because 6.60 mb (the pressure at the Pathfinder site) is above the triple point, the ice will melt when heated to 274 K. C: Because 6.00 mb (Mars' average pressure) is below the triple point, liquid water cannot exist under these conditions. Therefore, exposing the vapor to average Martian atmospheric conditions will cause it to undergo a phase transition directly to ice (deposition) Concept: A phase diagram shows the stability of each phase (solid, liquid, and gas) as a function of pressure and temperature. Equilibrium between two phases is indicated by phase boundaries, and crossing a phase boundary indicates a phase change. The triple point is the temperature and pressure at which all three phases exist in equilibrium.

q28

concept: An electron configuration accounts for all electrons held by an atom or ion, listed sequentially by shell and subshell in order of increasing energy. During cation formation, electrons are lost from the highest-energy filled shell/subshell first; during anion formation, electrons are added to the lowest-energy unfilled shell/subshell first.

q14

concept: Compounds may be described by their empirical, molecular, and structural formulae. Empirical formulae describe only the relative ratios of each type of atom, molecular formulae describe the actual numbers of each type of atom, and structural formulae describe the spatial arrangement of the atoms.

q16

concept: In all forms of beta decay, the mass number remains unchanged whereas the atomic number increases (β− decay) or decreases (β+ decay and electron capture) by 1. β− decay converts a neutron into a proton and emits an electron. β+ decay and electron capture convert a proton into a neutron (the opposite of a β− decay).

q15

concept: Ionic radii decrease in size across a period and increase down a group on the periodic table. The use of isoelectronic series is helpful in comparing cations, anions, and neutral atoms.

q8

concept: Lewis dot structures represent the distribution of bonds and valence electrons between atoms in a molecule, with the least electronegative atom in the center. A correct Lewis structure will show all valence electrons and complete octets for each atom, and yield the correct formal charge.

q17

first you have to find the half of the sample (y axis) and match it up with the time on x axis. Multiply that time by 1.5 to get the time after 1.5 lives have passed and match that up with the sample amount on the y axis. The ans choices are in M so you need to divide the sample of mmol by 2.0 mL to get M. You don't have to convert bc the ratio is mmol/mL which would be the same when its mol/L concept: A half-life is the time required for an amount of a given radioactive isotope to decrease by half. With each subsequent half-life interval that passes, the amount that remains decreases by half again, becoming exponentially smaller and eventually approaching zero. The molar concentration of a solution at a given time is determined by dividing the number of moles of analyte by the sample volume (in liters).

question 16

have to pick a buffer that is 1 pH unit away from the one given. The only ans that makes sense is MES bc it falls in the range of 5.3-7.3. All the other choices are out of range Concept: Buffers are mixtures of a weak acid or base and its corresponding conjugate salt that can resist changes in pH by neutralizing added hydronium or hydroxide ions. They are effective within 1 pH unit of their pKa. The pKa of a buffer is related to its acid dissociation constant (Ka) by the equation pKa = −log(Ka).

q52

just need to convert one of the celcius temps to get the range Kelvin = (°C + 273) = (−196 °C + 273) = 77 K concept: The Kelvin and Celsius scales differ only in the placement of the zero value. Therefore, a change of 1 degree Celsius is equal to the change of 1 Kelvin. A temperature on the Celsius scale can be converted to its equivalent on the absolute Kelvin scale by adding 273.15 to the Celsius temperature.

Q5

looking at the electron configuration you can see that it goes fro, 2p^6 to 3p^1 instead of going to 3s orbital after 2p^6. Thats why the electron was excited from the 3s orbital concept: Electrons typically occupy the lowest energy configuration (ground state), but an electron may achieve an excited state and temporarily jump to a higher energy orbital by absorbing energy equal to the energy difference between the two levels.

q16

metals are conductive concept: Elements can be broadly classified on the periodic table as metals, metalloids, or nonmetals. Due to many shared characteristics within each of these categories, the general traits of each type of element are often useful for qualitative comparisons between elements.

What is the difference between state and path function?

state function - Describe the physical properties of an equilibrium state. Are pathway independent. Pressure, density, temp, volume, enthalpy, internal energy, Gibbs free energy, and entropy. Path function - Functions whose value depends on the path taken to get between two states are called path functions. To understand the difference between the two, think of two cities, A and B, that are 100, k, m,100km away from each other along a straight line. Roads are rarely built in long straight lines, we may have two roads linking A and B, with lengths 150, k, m,150km and 200, k, m,200km. So, while the displacement between the two towns is the same for each route (it is a state function), the distance you need to travel to get from A to B depends on which road you take (it is a path function).

q3

table gives you bond enthalpy so you use that to figure out enthalpy of rxn which is: ∆H∘rxn = ∆H∘bonds broken - ∆H∘bonds formed Broken: 2 moles of H2 = 2(436) = 872 1 mole O2 = 495 Formed: 2 moles H2O = 4(499) = 1996 872 + 495 - 1996 = -629 Educational objective: Bond enthalpy is the energy needed to homolytically break 1 mole of a given type of bond between two atoms in the gas phase at 298 K. According to Hess' law, a reaction can be assessed as a series of endothermic bond-breaking and exothermic bond-forming steps, and the heat of the reaction is equal to the sum of the enthalpy changes for each step.

q52

this q refers to electron affinity. Cs has the lowest electron affiniry in group 1 and is least likely to accept an electron concept: Electron affinity is the change in energy associated with adding an electron to a neutral atom in the gas state. Values with a more negative magnitude indicate elements that more readily accept the addition of an electron. With some intermittent exceptions, electron affinity tends to become stronger (more negative) moving left to right across a period and tends to become weaker (less negative) moving down a group on the periodic table.

q48

to find neutrons you just subtract mass number from the atomic number concept: Isotopes are designated by their mass number, which is the sum of the number of protons and neutrons in the nucleus. The number of neutrons in an isotope can be determined by subtracting the element's atomic number (number of protons) from the mass number. The number of electrons in a neutral atom is equal to the number of protons.

q 1

You see the concentratin at equilibrium is 6 mol/L So you convert 750 ml to .75L and multiply it by 6mol/L to get mols of the sample which is 4.5 mol. concept: Equilibrium is achieved when two opposing chemical reactions occur simultaneously at the same rate such that the concentrations of the chemical species become constant. Equilibrium reaction rates are equal, but the equilibrium concentrations of chemical species may be unequal.

question 18

A dipole moment occurs when positive charge accumulates at one end of a molecule and negative charge accumulates at the other. The greater the difference in charge at each end of the molecule, the greater the dipole moment. A molecule may have multiple polar bonds, each with its own dipole moment. If the individual moments all point in the same direction, the overall moment is increased. If individual moments point in opposite directions, they cancel each other and the molecule as a whole has no dipole. For this reason, symmetrical molecules generally do not have net dipole moments and are considered nonpolar. As shown in Figure 2, oxalate can take on different forms depending on the pH to which it is exposed. When the pH < 1.25, the molecule is fully protonated and becomes oxalic acid. Resonance forms show that individual regions of this molecule have dipole moments. However, this form of the molecule is symmetrical so the dipoles cancel, giving no net dipole moment. Similarly, while the molecule is fully deprotonated at a pH > 4.14, it is still symmetrical and has no net dipole moment in this form either. concept: Molecules have a net dipole moment (separation of charge) when the individual dipoles within it do not cancel each other. Symmetrical molecules typically do not have net dipole moments. Protonation and deprotonation can change whether a molecule is symmetrical and can induce or remove a dipole moment.

q3

A disproportionation reaction is a redox reaction in which both the oxidation and the reduction occur to atoms of the same element. You have to go through every equation to see which molecule goes through both oxidation and reduction. After the first reaction, you can tell that one is not a disproportionate so you can eliminate choices B and D. Choice C includes the second reaction so you just have to check that one. Easier way to do this would be by seeing if theres twice of one molecule in the products side bc you need it to appear twice for oxidation and reduction to happen to the same atom concept: In a disproportionation reaction, the same element (at a given oxidation state) undergoes both oxidation and reduction, with some of the atoms being oxidized and other atoms of the same element being reduced. Comparing the oxidation number for each element in the reactants with the oxidation number for the same element in the products enables the identification of a disproportionation.

q14

According to Hess law, the overall enthalpy change ΔH° of a reaction is equal to the sum of changes in enthalpy of each component. Therefore, the enthalpy of formation for NaOH can be calculated by subtracting the enthalpies of bleach, ammonia, and dichloramine from the overall enthalpy of the reaction (−55 kJ/mol), applying the following: 1) The signs of the reactants must be changed. Table 1 gives enthalpies of formation, but ammonia and bleach are consumed, not formed. Therefore, they contribute +350 kJ/mol and +45 kJ/mol of enthalpy, respectively. 2) The standard enthalpies of formation for each component must be multiplied by their stoichiometric coefficients. In the case of bleach, two molecules are consumed per reaction, so the standard enthalpy must be multiplied by 2. Similarly, two molecules of sodium hydroxide are formed per reaction, and these must be accounted for. concept: Hess's law states that the overall enthalpy change of a reaction is equal to the sum of the enthalpies of formation for each component. Because reactants are consumed rather than formed, the signs of their enthalpies must be changed. Each component must be multiplied by its stoichiometric coefficient to account for the total number of molecules consumed and formed in each reaction.

Q20

According to the passage, one of the advantages of NiCd batteries is that they have a higher energy-to-weight ratio than lead storage batteries. Even though lead-acid batteries have a higher E°cell, NiCd batteries have more energy per unit mass and are smaller in size. Therefore NiCd batteries have a higher energy density. Because lead batteries have a higher E°cell than NiCd and the amt of energy a cell provides is directly proportional to the potential E°cell, lead batteries produce more energy in absolute terms than NiCd batteries concept: Energy density is the amount of energy produced by a battery per unit of mass. It is used to compare the energy output of batteries, or the work done by the battery. The electromotive force that a battery produces on discharge is the amount of work done per unit of charge.

Q26

An LD50 gives the concentration of a substance that is necessary to kill 50% of the group being tested. Comparing the experimental results for the same species shows that the solution containing the HCN form of cyanide gave the lowest LD50 of the three forms of cyanide delivered. This indicates that HCN required the smallest dose to kill 50% of the rabbits and rats tested. The lower LD50 for HCN suggests that this neutral molecule may diffuse more readily across the gastric lipid membrane than the charged ionic cyanide salts Concept: An LD50 value gives the concentration of a substance necessary to kill 50% of a group being tested. Therefore, a small LD50 requires less of a compound for a lethal dose than a compound with a higher LD50. These values can be used to evaluate the relative toxicity of various compounds and to infer differences in how the compounds are metabolized.

q20

Again manipulation of the ideal gas law. This one is talking about Charles's law which says that volume and temperature are directly proportional to each other when P and n are held constant: V∝T concept: Charles' law states that there is a direct relationship between the volume and the temperature of a fixed number of molecules of an ideal gas at constant pressure (ie, as temperature increases, volume increases).

q11

Again use POE and if you're btw two choices move on to the next decay. Fastest way to find the answer concept: In α decay, the mass number of an atomic nucleus decreases by 4 units and the atomic number decreases by 2 units. In all forms of beta decay, the mass number of an atomic nucleus remains unchanged but the atomic number increases (β− decay) or decreases (β+ decay) by 1 unit.

q24

Alkaline earth metals are group 2A metals and they form increasingly vigorous recations when forming ionic bonds with nonmetals. This requires the valence electrons of the metal atom to be removed and transferred to the nonmetal atom. Of the four properties previously discussed, only ionization energy quantifies how easily electrons are removed from an atom. Therefore, ionization energy best explains the reactivity trend for Reactions 1-3. Moving down the alkaline-earth metal column, ionization energy decreases. This makes removing an electron more favorable and increases reactivity. Educational objective: Ionization energy is the energy required to remove an electron from an atom. Because of the associated ionization and electron transfer involved in forming ionic bonds, the reactivity of atoms forming ionic compounds increases as the ionization energy decreases.

q22

All of the ones are diamagnetic bc there are no unpaired electrons concept: Electrons fill the shells and subshells of an atom in order of increasing energy according to Hund's rule and the Pauli exclusion principle. Unpaired electrons in the electronic configuration of an atom or ion result in paramagnetism, but a configuration without unpaired electrons results in diamagnetism.

Q24

All the variables are given. Just had to do the math out concept: A water-soluble salt formed from the conjugate base of a weak acid dissociates fully in an aqueous solution. Because the salt contains the conjugate base of a weak acid, the resulting solution often exhibits a basic pH with a base dissociation constant Kb that correlates to the corresponding weak acid.

q18

Basically apply le chatelier's principle here. If you want to stop the rxn from going forward you either want to take away reactants or add more products. Choice A is adding a product and thats why its an effective way to stop the rxn concept: Le Châtelier's principle states that changes to a system that move it away from equilibrium will cause the system to adjust and come back to equilibrium. The addition of reactants or removal of products results in product formation, and the addition of products or removal of reactants results in reactant formation.

question 18

An electron balance must be present between the half-reactions to achieve the balanced net reaction from the sum of the two half-reactions: Electrons present as reactants in one half-reaction should appear as products in the other half-reaction. Identical species appearing as reactants in one half-reaction and as products in the other cancel and are not part of the net reaction expression. Based on these principles, an unknown half-reaction can be found by adding the balanced net reaction to the inverse of the known half-reaction. Species in the inverse known half-reaction that do not participate in the discharge will cancel with the balanced net reaction, leaving only those species present in the other half-reaction occurring during discharge (basically cancel out whatever cancels) concept: The balanced net reaction for an electrochemical cell may be found by taking the sum of the half-reactions, written to be consistent with the direction of electron flow within the cell. Half-reactions may be scaled stoichiometrically to achieve an electron balance. Species matching in type and number on both sides of the summation are not included in the net reaction.

question 35

As described in the second paragraph of the passage, when heme b (Figure 1) axially binds with oxygen and a globin histidine, the Fe2+ has three ligands (1 porphyrin, 1 histidine, and 1 O2) that together form six coordinate covalent bonds: four to amines in the porphyrin, one to histidine from the globin protein, and one to an O2 molecule. Therefore, Fe2+ and its ligands form an octahedral arrangement in which the six coordinate bonds are all at approximately 90° angles from each other. concept: Coordinate covalent bonds tend to maximize geometric space around a central atom. Transition metals generally form complexes with two, four, or six coordinate bonds. Therefore, the geometry of the coordinate covalent bonds will typically be linear (two bonds), tetrahedral or square planar (four bonds), or octahedral (six bonds).

question 19

Because the measured potential for the cell is positive (E = 0.089 V), the ion concentrations must have a ratio of [X]anode/[Y]cathode that is less than one so that the logarithmic term of the Nernst equation will be negative. A negative logarithmic value multiplied by the negative constant in the equation results in a positive value for E. Therefore, [Ni2+] is greater at the cathode, and the electrons in this cell will flow from the anode to the cathode. concept: Concentration cells use the same electrode material and ionic solution at both the anode and the cathode but operate by a concentration difference between the cells, which produces a difference in potential. Electrons will migrate from the anode (fewer cations) to the cathode (more cations) until the concentrations equalize.

Question 15

As seen in Figure 1 of the passage, the solubilities of CuCl2 and CuSO4 salts both decrease as the temperature decreases, but the solubility of CuSO4 decreases to a much greater extent. Consequently, when attempting to separate a mixture of CuCl2 and CuSO4 salts at 90 °C, cooling the mixture to 5 °C promotes the precipitation of the CuSO4 while the CuCl2 tends to remain in solution (Number III). To further decrease the solubility of the CuSO4 that would still persist in solution, the common ion effect could be applied by adding Na2SO4 to the solution (Number II). CuSO4 and Na2SO4 have the SO42− ion in common; therefore, adding Na2SO4 will increase the SO42− concentration, which will shift the solubility equilibrium toward CuSO4 and cause the CuSO4 to selectively precipitate out of the mixture. concept: The temperature dependence of solubility and the common ion effect from relevant ions on the solubility equilibrium can be used to assist in selectively precipitating compounds during separations of mixtures.

question 29

As shown in Figure 3, the y-axis indicates cell response, and it is evident that D-amino acids elicit only a small response, if any, from cells; of the D-amino acids, only D-Ala caused a noticeable cell response. Because responses are caused by tastant binding, it can be concluded that a majority of D-amino acids do not bind effectively to T1R1+T1R3 GPCR cells. concept: Interpretation of graphic data requires an understanding of the underlying mechanisms. Cells may have altered responses to stimuli for a number of reasons. In this case, amino acids induce a response by binding to a receptor, so failure to induce a response indicates a lack of binding.

question 36

As stated in the last sentence of the passage, ZPP has an excitation maximum of 425 nm and an emission maximum of 595 nm whereas PPIX is excited at 407 nm and emits a photon at 625 nm. Because ZPP can be excited by a longer wavelength than PPIX, ZPP requires less energy to excite its electrons from the ground state. concept: Electromagnetic radiation at a certain wavelength has photons of a particular energy. Energy is inversely proportional to wavelength.

q53

Asking which cation can displace K+ ion after stating that metal ions close to the size of K ION can displace it so you had to knwo the ionic radii trend and it increase down a group but all the choices were group1A elements anyways Educational objective: For ions having the same charge, ionic radii tend to decrease across a period and increase down a group on the periodic table. Ions of adjacent elements tend to be closer in size than those farther away.

q7

At higher temperatures, the molecules move more quickly and collide more often and with more energy. Therefore, an increase in temperature will increase the rate of a reaction by providing more molecules with the required activation energy. The rxn btw H and O is thermodynamically favorable but its slow at room temp bc of a high activation energy. So, the spark releases heat into the system, raises the temperature and therefore raises the KR of the molecules. These molecules are then able to react exothermically, releasing more heat into the system and providing more molecules with sufficient energy to react. concept: Molecules must collide with sufficient energy, known as the activation energy, for a reaction to occur. Increased temperature increases the kinetic energy of the molecules in a system, and therefore increases the rate of a reaction.

q20

Because light energy is inversely proportional to the wavelength, the line emission at 410 nm (the shortest wavelength in the line spectrum) must correspond to the electron originating in the highest-energy orbit. Therefore, the energy diagram depicting the transition of an electron from n = 6→2 (the highest energy emission) is associated with the shortest spectral line wavelength (410 nm). shortest wavelength (410 here) - means largest energy so from highest energy level and number of lines on emission spectra determine energy levels, and energy is always emitted when coming down from excited state so not during absorption but emission Educational objective: In the Bohr model of the atom, electrons move around the nucleus in fixed circular orbits at particular intervals. Electrons in orbits farther from the nucleus have higher energy. Energy equal to the difference between two orbits is absorbed by an electron moving to a higher orbit and is emitted by an electron moving to a lower orbit.

question 40

Because the glowing splint burned more brightly in the gas that was produced when the MnO2 was added, one of the products of the decomposition of H2O2 is O2 because in the passage they say O2 is produced when something burned more brightly. From this, it can be inferred that the other product is H2O. concept: Reaction types follow a general pattern and allow for the prediction of the products of a reaction that can then be verified in the laboratory.

Colligative properties

Boiling Point Elevation: If you add solute to a liquid it gets in the way of water trying to evaporate, therefore its harder to raise vapor so boiling temperature increases bc its harder to evaporate. So there's an increase in BP (look at vapor pressure to learn more) Freezing Point Depression: If you add solute to a liquid it will get in the way of the regular crystal structure, making that regular ice structure LESS STABLE. Meaning you need to cool down the water even more to make it happen so the freezing point lowers. Vapor Pressure Lowering: So adding solutes to liquid will lower the vapor pressure of the liquid bc it will block the water molecules from evaporating/interacting with one another as effectively so that means that more heat is required to equal its vapor pressure to the atmospheric pressure so that's why there's an increase in BP Osmotic Pressure: Water follows solute, so if there's solute on one side of a permeable membrane and it only allows wtaer through then water will flow towards the higher concentration of solute because there is less water in that area

q43

Boiling point is when the temperature is high enough to the point where the vapor pressure is equal to the atmospheric pressure. In the psg more solute is added to the briny water. Briny water will therefore need to be heated to a higher temperature for its vapor pressure to reach the same ambient pressure and begin to boil. Therefore, the boiling temperature (boiling point) of briny water is higher than that of pure water under the same atmospheric conditions. concept: The boiling point temperature is the temperature at which the vapor pressure is equal to ambient pressure. The addition of solute lowers the vapor pressure of a solution at all temperatures, and therefore raises the temperature required for the vapor pressure to become equal ambient pressure and begin to boil (the boiling point).

q23

Daltons law: Total pressure is equal to the sum of the partial pressures. q stem gives you the total pressure and the percentage of the two partial pressures. To find the actual number you need to just multiple the total pressure (3 ATA) with the percentage of the partial pressures. O2 is 80% so: .8 times 3 = 2.4 ATA Concept: Dalton's law of partial pressures states that the total pressure of a mixture of gases is equal to the sum of the partial pressure of each of the individual gases. The partial pressure of an individual gas can be found using the mole fraction of the gas multiplied by the total pressure.

question 58

D2sp3 means there are 2+1+3 = 6 bonds to the central atom so it has to be SF6 an octahedral Concept: In hybridization, the number of hybrid orbitals that form must be equal to the number of atomic orbitals that hybridize. The sum of the number of bonding regions and the number of nonbonding electron pairs around the central atom in a compound must be equal to the number of hybrid orbitals employed by the central atom.

question 7

During the formation of copper(II) glycinate (Figure 1), glycine displaces acetate in Cu(C2H3O2)2 because stronger Lewis bases can displace weaker Lewis bases as ligands within a complex. Stronger Lewis bases tend to be those that have lone pair electrons on atoms with a net charge and/or a lower electronegativity. As a ligand, glycine has an available pair of electrons to donate on both the oxygen atom in the carboxylic group and on the nitrogen atom of the amino group, and both sites in the structure of glycine function as Lewis bases (electron pair donor) in the reaction shown in Figure 1. This is indicated by the formation of the Cu−N and Cu−O coordinate bonds in the complex. concept: Within coordination complexes, a ligand acts as a Lewis base, and the metal center acts as a Lewis acid. Ligand structures that can form more than one coordinate bond by donating more than one pair of electrons have more than one site that can function as a Lewis base.

q56

Given that reactivity is driven by the tendency of atoms to gain, lose, or share electrons to achieve a more stable electronic configuration, noble gases tend to be unreactive (except in extreme cases) as they already possess an energetically favorable configuration. concept: Reactivity is driven by the tendency of atoms to gain, lose, or share electrons to achieve a more stable electronic configuration (ie, a full valence shell). Noble gases are generally unreactive because they already have a full valence shell of electrons.

q13

Got it wrong bc i didnt realize that -1 to +1 = loss of 2 electrons bc the number is becoming positive by 2 values concept: Oxidation numbers track the transfer of electrons between atoms during a redox reaction. An increased oxidation number corresponds to a loss of electrons (oxidation), whereas a decreased oxidation number results from gaining electrons (reduction). A change of one unit occurs for each electron transferred.

q36

Had to start with 44.8L ammonia and they are asking to find moles of O2 so you can set up stoichiometry Educational objective: Under conditions of standard temperature and pressure (STP), which is defined as 0 °C and 1 atm of pressure, 1 mole of gas occupies a volume of 22.4 L. Mole ratios from balanced chemical reactions can be used to relate the moles of a given gas to the moles of other reaction species

q26

Had to substitute Ca in for M for rxn 2 to find the equation for the calcium metal reaction and then do the math Educational objective: At standard temperature and pressure (STP), gases in reactions can be related to the molar volume of 22.4 L/mol. Mole ratios from balanced chemical reactions can be used to relate the moles of the gas to the moles of other reaction species. The mass of a given number of moles of a particular species can be found using its molar mass.

question 31

Each orbital is associated with a different energy level, and the Aufbau principle states that electrons will fill the lowest energy levels first. The periodic table is arranged to reflect the relative energies of each orbital type and, therefore, the order of electron orbital filling. The orbitals are arranged into different blocks on the periodic table: s orbitals are in the first two columns (except helium), the p orbitals are on the right, the d orbitals are in the middle, and the f orbitals are at the bottom. Therefore, the order of orbital filling can be determined simply by finding an atom's position on the table. Iron is within the 3d orbital section, which comes after the 4s section, so 4s is filled first. concept: The Aufbau principle states that low energy orbitals are filled first. The periodic table can be used as a guide to orbital filling. Each block on the table represents different orbital types, with s orbitals in the two left columns, p orbitals in the six right columns, and d orbitals in the middle, and f orbitals at the bottom.

q15

Entropy is a measure of the amount of disorder in a system. The psg gives a negative value for ΔS° of the overall reaction (−150 J/mol·K). So, over the course of the reaction, the entropy of the system decreases and the order increases. Educational objective: The change in entropy (disorder) of a system over the course of a reaction is expressed by ΔS° and must be measured empirically. A positive ΔS° corresponds to an increase in entropy (more disordered) whereas a negative ΔS° corresponds to a decrease in entropy (more ordered).

question 3

Equation 1 is a double replacement reaction because the bicarbonate anion (HCO3−) exchanges the sodium counter ion for an acidic proton from hydrochloric acid (forming carbonic acid, H2CO3), and the hydrochloric acid exchanges the acidic proton for the sodium ion (forming sodium chloride, NaCl). concept: Chemical reactions can be categorized broadly into five main types: combination, decomposition, single replacement, double replacement, and combustion. Double replacement reactions involve the exchange of the bonding partners of two reactant compounds, forming two new product compounds.

question 25

Every additional bond formed between two atoms after a sigma bond is a pi bond. Pi bonds are created by the sideways overlap of p orbitals along a plane perpendicular to (ie, above and below) the internuclear axis. Because the overlap in electron density is not as efficient as for sigma bonds, pi bonds exist in a higher energy state and are not as stable as sigma bonds. As a result, they require less energy to be broken than sigma bonds (ie, they have a smaller dissociation energy) (Number I). Although individual pi bonds are weaker than sigma bonds, a double bond is composed of both a sigma and a pi bond, and therefore is stronger than a single bond (Number II). III. Because the pi bond in C=O is higher in energy than the sigma bond, it is less stable. Concept: Sigma bonds are lower in energy and more stable than pi bonds, and therefore have a higher dissociation energy. Sigma bonds form first between two atoms, and every bond formed thereafter is a pi bond. Although individual pi bonds are weaker than sigma bonds, a double bond is composed of both a sigma and a pi bond, and therefore is stronger than a single bond.

question 12

FC = should - has concept: A formal charge is the charge assigned to an atom based on an accounting method which assumes that the bonding electrons between two atoms are shared equally. Formal charge equals the group valence of an atom minus the number of nonbonding electrons and minus half of the number of bonding electrons.

q42

FON = hydrogen bond Hydrogen bonding is a strong intermolecular force that occurs between a hydrogen atom on one molecule and an electronegative atom with a lone electron pair on another molecule. Carbon is a small atom but not sufficiently electronegative to participate in hydrogen bonding. On the other hand, bromine is electronegative but too large to participate in hydrogen bonding. Therefore, C2H4Br2 is unable to form hydrogen bonds with water. concept: Hydrogen bonding occurs when a partially positive hydrogen atom in one molecule is attracted to the partial negative charge on an electronegative atom in another molecule. Atoms involved in hydrogen bonding must have small atomic radii, and are essentially limited to fluorine, oxygen, and nitrogen.

q58

Figure 2 shows that ΔG is negative for the formation of Compound 2 from the intermediate, Keq > 1 for the conversion Concept: The Gibbs free energy ΔG° and the equilibrium constant Keq of a reaction are related through the gas constant R and absolute temperature T by the equation ΔG° = −RT lnKeq. Negative values of ΔG° indicate that Keq > 1 and that the equilibrium favors product formation whereas positive values of ΔG° indicate that Keq < 1 with an equilibrium that favors the reactants

q26

Had to use avogadro's law here and to do that you need to know moles first. The masses of both N2 and O2 are equal (1.5g each) so you know that the element with the lower molar mass would produce more moles bc the ratio has 1.5/molar mass so molar mass is lower for N2 (28 g/mol) vs O2 (32 g/mol) so N2 has more moles and occupies a larger volume Educational objective: Avogadro's law states that the volume occupied by a gas is directly proportional to its number of moles if the temperature and pressure are constant. When two gases at the same temperature and pressure are compared, the gas with more moles occupies the larger volume.

question 30

Figure 3 shows that some molecules exhibit a greater overall response to tastant than others, as well as a greater difference in response between tastant and tastant + IMP. The increase in the ratio of the response with IMP to the response without IMP is greatest for Asp. The degree of potentiation by IMP for a tastant molecule is defined by the relative increase in response upon addition of IMP. Therefore, IMP most effectively potentiates Asp. (Choices A and B) Figure 3 shows that the ratio of the response of tastant + IMP to tastant alone for each molecule is different. Therefore, IMP potentiates different tastant molecules to different degrees. (Choice C) Although the difference in responding cells for Thr with and without IMP (~120 cell difference) is much greater than for Asp (~60 cell difference), the ratio of the response for tastant + IMP relative to the initial condition is less for Thr (~4:1) than for Asp (~60:<1). Concept: Drawing conclusions from experimental results requires careful analysis of the experimental data. The degree of potentiation, or a change in a system's response to a stimulus, is best measured as a ratio. This ratio shows the percent change in a signal's intensity in the presence of a potentiator.

q25

For a reaction at equilibrium, the law of mass action states that an equilibrium constant Keq can be expressed as a ratio of the molar concentrations of the products over the reactants, each raised to the power of its respective balanced reaction coefficient. BUT for a rxn where one of the reactants is also the solvent, the solvent species is omitted from the Keq expression. The concentration of H2O is omitted from Keq expression bc water is both a reactant and solvent. As the solvent, the number of water molecules is significantly greater than the number of all other species in the solution; the HC2H3O2 is dilute and dissociates only to a small extent. Therefore, in relative terms, the [H2O] (55.5 M) is essentially constant and is omitted Educational objective: Applying the law of mass action, an equilibrium constant Keq can be expressed for a reaction as a ratio of the molar concentrations of the products over the reactants, each raised to the power of its respective balanced reaction coefficient. Because the concentration of the solvent is much greater than the concentration of all other species in the solution, the solvent is relatively constant and is omitted from the Keq expression

Q15

For an electrolytic cell, this value is negative, indicating that the oxidation-reduction reaction is not spontaneous. During the charging phase of a rechargeable battery, an external potential is applied to force the oxidation-reduction reaction to proceed in a non-spontaneous direction. Because Eocell for this battery is a negative value (−2.0 V), the cell is functioning as an electrolytic cell. concept: When a battery is charging, it functions as an electrolytic cell because an external potential forces the oxidation-reduction reaction to proceed in a nonspontaneous direction. This makes the electromotive force for the cell a negative value.

q22

Had to use the PV=nRT equation and find the mols at each pressure and then subtract to get the mol that is added to the chamber. Or you could just find the change in pressure and just solve for n. concept: The ideal gas law is a formula that describes the number of moles of an ideal gas under given conditions of pressure, volume, and temperature related via the universal gas constant.

Q12

For the dissociation of NaC9H7O4, which releases one Na+ ion and one C9H7O4− ion, Ksp = [Na+][C9H7O4−]. Because 1 mole of Na+ and 1 mole of C9H7O4− are released into solution for each mole of NaC9H7O4 that dissociates, [Na+] and [C9H7O4−] are always equal. Instead of doing sqrt of 34.9, easier and faster way to find the answer would be to think what the sqrt of 36 is. Which is 6. So that means mean sqrt 34.9 would be a little less than 6 concept: The solubility product constant Ksp represents the limit of solubility for a compound, and it is defined as the product of the molar concentrations of the dissolved species each raised to the power of their respective balanced equation coefficients. This relationship can be used to find molar concentrations of ions in saturated solutions.

Question 2

For this question you had to compare the electronegativity difference between the atoms. Comparing NaBr and MgBr2, NaBr has a higher electronegativity difference (and higher ionic character) because Na is further to the left of Br on the periodic table than Mg. Similarly, NaCl has a higher ionic character than MgCl2. Of the two higher-ranking sodium salts, NaCl has the greatest electronegativity difference (ionic character) as Cl is above Br in the halogens column. Concept: Electronegativity trends on the periodic table are useful to assess relative differences in electronegativity between elements. The difference in electronegativity between two bonded atoms is directly proportional to (and indicative of) the degree of ionic character of the bond between the atoms.

question 11

For this question you have think le chateliers principle and you have to think acid base equilibrium. As discussed in the passage, the solubility of CuF2(s) in water is low because the equilibrium (Reaction 1) favors the undissolved salt. However, the fluoride ion is mildly basic, and this property can be exploited to disrupt the equilibrium because bases react with acids. Therefore, if nitric acid (source of H+) is added to the solution, some of the F−(aq) will be used by the acid to form HF, and the concentration of F−(aq) will initially decrease. The decrease in F−(aq) will disrupt the CuF2(s) equilibrium, causing a shift toward the products to form more F−(aq). Consequently, more CuF2(s) will dissolve and the Cu2+(aq) concentration will increase on adding dilute nitric acid. concept: Changing the pH of a solution can enhance the solubility of salts that have an acidic or basic ion. This occurs by applying Le Châtelier's principle to disrupt the solubility equilibrium and shift the reaction toward the products.

question 7

For this, they give you the OH concentration, so you have to find the H+ concentration by dividing 1x10^-14/1x10^-6. This gives you the H+ concentration which is 1x10^-8. pH = -log(H+). Since the H+ concentration is 1x10^-8 so the pH is 8 concept: The [H3O+] and [OH−] concentrations are related by the self-ionization constant Kw of water according to the expression Kw = [H3O+][OH−] = 1.0 × 10−14 M2. Using a logarithmic approach, pH is related to [OH−] by the relationship pH + pOH = 14, where pOH = −log[OH−].

Q1

From the question I should have realized that they gave the concentrations of a weak acid and its conjugate base. So, to find the pH I need to use the handerson hassalbach equation Since the pH works out to be 5.8, it does fall within the pH range for urine which is 4.5 to 8.0 (given in the passage) concept: A buffer resists changes to pH. A buffer system is comprised of a weak acid (or weak base) and its conjugate. The pH of a particular buffer system can be found using the Henderson-Hasselbalch equation.

q24

Graph interpretation Two of the assumptions made by the ideal gas law are that the volume of the individual particles of gas is negligible, and the individual particles of gas do not interact with each other. These assumptions are adequate for most gases at low pressure. However, at higher pressures the individual particles are closer together, resulting in an increased gas density. An increased density results in a greater likelihood that the gas particles will interact with each other in a nonelastic way. Furthermore, at extremely high pressures, the combined molecular volume occupied by the gas particles becomes significant relative to the volume of the container. These factors lead to deviations from ideal behavior in real gases. concept: At higher pressures, deviations from ideal gas behavior occur because the density of a gas increases enough that the individual gas particles are more likely to interact in a nonelastic way, and the molecular volume occupied by the gas particles becomes significant relative to the volume of the container.

q7

Group 16 is the only group that meets all those criteria mentioned in the question. I got confused with the metalloids part. Metalloids separate the metals and the nonmetals so everything on the right of the metalloids are nonmetals and vice versa O is the diatomic gas S and Se are the nonmetal solids Te is a metalloid Po is a radioactive element (some say Po is also a metalloid so im assuming that the metalloid labels change) concept: Elements are arranged into groups on the periodic table based on their electronic configurations. Elements within the same group have the same number of valence electrons (similar chemical properties) but can have very different physical properties.

Q6

Had to do mol to mol conversion from oxygen to nitrogen using the equation and then convert mols of nitrogen to grams of nitrogen using the MM concept: The molar mass (the amount of mass in 1 mole of a compound) permits conversions between a given amount of mass and the number of moles that it contains. Stoichiometric mole ratios from a balanced reaction equation can be applied to determine the number of moles of one chemical species relative to the moles of another chemical species in a reaction.

q30

Had to know that a catalyst does not affect the heat of the rxn, it only affects the rate of a reaction. Therefore, the reaction using Fe(NO3)3 as the catalyst will result in the same change in temperature as the reaction using NaI as a catalyst, assuming the same amount of H2O2 is used. Educational objective: A catalyst increases the rate of a reaction by lowering the activation energy needed for the reaction to occur. A catalyst does not change the amounts of the products produced or the enthalpy (heat) of the reaction.

q10

Had to know what Pauli's exclusion principle is: states that each orbital within a subshell can hold a maximum of two electrons, but two electrons in the same orbital must have opposite spins. so anything with same spins is eliminated and anything with electrons more than the subshell can hold is eliminated knew electrons in each subshell: s = 2 p = 6 d = 10 f = 14 Educational objective: In an electron configuration, subshells are labeled by type as s, p, d, or f, which contain one s orbital, three p orbitals, five d orbitals, and seven f orbitals, respectively. According to the Pauli exclusion principle, each orbital within a subshell can hold a maximum of two electrons, but two electrons in the same orbital must have opposite spins

q12

Had to look at fig 2 for conditions given in Qstem so look at grey line bc they say the specific atm of H2 and O2 and then find the rxn rate which is 6 and convert to grams of H2O **don't forget the time of 1 hr in qstem** Educational objective: Given a constant reaction rate, the amount of product formed during a reaction can be calculated as the rate multiplied by the time in which the reaction proceeds. Units may need to be converted between mass and moles

q6

Had to look at the ΔH and ΔS form psg and you know a negative linear slope in fig 1 is indicative of +ΔH and also from fig 1 equation you get a +ΔS. The positive ΔS means that entropy increases and if ΔH is positive that means its an endothermic rxn which means that energy is put into the reactants to make products which means the reactants are are at a lower energy than the products energy Educational objective: Endothermic reactions have a positive ΔH° and absorb energy (heat) from the surroundings as the reaction proceeds because the products have higher energy than the reactants. An endothermic reaction can result in either an increase or a decrease in entropy, depending on the change in the extent of disorder within the system.

q51

Had to use the standard gibbs free enrgy equation: ∆G°=−RT ln Keq Keq = [products]/[reactants] look at the math --> Educational objective: The equilibrium constant Keq for a reaction is expressed as a ratio of the molar concentrations of the products over the reactants, each raised to the power of its respective balanced reaction coefficients. The standard Gibbs free energy ΔG° and the Keq are related via the gas constant R and absolute temperature T by the equation ΔG° = −RT ln Keq.

q58

Halogens with a smaller atomic number have a smaller atomic radius and form shorter, stronger H−X bonds that are more difficult to ionize. Smaller halogens are more electronegative and more readily attract electrons in the H−X bond toward itself, creating a polar covalent bond. The stability of the conjugate base of the hydrogen halides (X-) also affects acidity. The conjugate base formed from a smaller halogen is less stable because it has a smaller atomic radius (less surface area) and cannot spread out (stabilize) the negative charge as well as halogens with larger atomic radii (greater surface area). A short, strong H−X bond that is difficult to ionize in aqueous solution indicates a weak acid. Concept: The acid halides are all strong acids except for H−F. Fluorine is the most electronegative and has the smallest atomic radius of the halides, resulting in strong bond formation.

q5

Have to know that adding a catalyst or an enzyme affects the reaction rate but DOES NOT affect: equilibrium (Ka) enthalpy entropy temperature A catalyst is a chemical species that is added to a reaction to increase the rate of the reaction by stabilizing the transition state and decreasing the activation energy of the reaction. A catalyst does not change the amount of products produced and does not change the relative energies of the reactants and products Educational objective: Thermodynamic values indicate if a reaction is spontaneous or not, but they do not indicate the rate of a reaction because thermodynamic values are state functions that are independent of the reaction pathway.

q2

Have to know the linear equation of a line: y=mx+b and that the Van't Hoff equation is the same as the linear equation and Qstem gives you -4911 for slope (m) and 5.4 y-intercept (b). You have to solve for delta H and delta S using equation 1 and the substitute into the delta G equation. Look at picture. (Choice C) incorrect: This is the answer obtained if the terms for the slope and the y-intercept (−4911/R and 5.4/R) from Equation 1 are substituted into the Gibbs free energy equation without first solving the respective terms for ΔH° and ΔS°. Educational objective: The Gibbs free energy is a state function that describes the spontaneity of a reaction. It is related to the temperature, enthalpy, and entropy of the reaction via the equation ΔG° = ΔH° − TΔS°, where T is given in Kelvin.

question 57

I drew out the shape of hydrogen sulfide which is H2S and S has 6 valence electrons which means it has two single bonds to H and two lone pairs meaning it will look like water. Also, looking at the table you can see that it has the same formula as H20 and S and O are in the same group so the molecular geometry must be the same Concept Atomic orbitals can combine into hybrid orbitals, which adopt an electron geometry that maximizes the separation and minimizes the repulsion between the electron-dense orbital regions. The molecular geometry (shape) of a molecule depends on the electron geometry and number of nonbonding electron pairs of the central atom.

question 10

I got this wrong because I didn't read the answer choice correctly. Thought they said cis first but they said trans first. You get the modes based on the peaks on the graph (picture). Comparing the number of vibration modes for each isomer to their respective far-IR spectra, the cis isomer is seen to display four peaks (in addition to the skeletal vibration peak at 385 cm−1); however, the trans isomer displays only two peaks along with the skeletal vibration peak. This indicates that two of the four vibrational modes of the coordinate bonds in the trans isomer are not IR-active. concept: Vibrational modes that do not produce a net change in a dipole are not IR-active and will not produce an absorbance peak in an IR spectrum.

Q2

INDICATORS DO NOT CHANGE THE PH OF THE SOLUTION!!! Indicators are used in acid-base titrations to determine the endpoint of a titration. A good indicator should change color close to the equivalence point, the point at which the stoichiometric amount of titrant needed to consume all of the acid or base in the solution has been added. Indicators do not participate in the reaction.

question 10

If a solution contains two salts that have an ion in common, the common ion supplied by one salt produces a common ion effect that stresses the product side of the equilibrium of the other salt and shifts the equilibrium to the left. Accordingly, for both salts the solubilities decrease. You have to use the ksp value given to set up the equation concept: In solutions containing two salts that have an ion in common, the common ion supplied by the one salt produces a common ion effect that shifts the equilibrium of the other salt to the left and decreases its solubility.

question 5

In Experiment 2, only 80% of the initial amount of oxytocin monoacetate was recovered from the sample with 10 mM Zn2+ additive after being stored for 4 weeks at 50 °C (Figure 2). You had to use the 100 (original) to 80 (recovery) ratio in the mol to mol conversion and also the concentration of oxytocin concept: Solution concentration can be expressed in terms of mass per unit volume, which can be used as a conversion factor to find the volume of solution that contains a given amount of mass. Conversions between the moles and mass of a substance can be made using the molar mass.

q13

In P4 they state the concentration and vol of LaCl3 that they used so you had to convert to IONS Educational objective: A mole is the amount of a substance that contains as many particles, ions, or atoms as the number of atoms in 12 g of 12C. Moles of a substance can be converted to ions (or particles, molecules, atoms) using Avogadro's number, 6.02 × 1023.

question 3

In a buffer solution adding more acid will decrease the pH and adding more base will increase the pH Exp 1, citrate buffer is made from citric acid and sodium acetate (conjugate base). Adding more of the conjugate base will increase the pH of the solution. The passage states that oxytocin is stable at pH 4.5 and the figure in the passage shows that at higher pH the change recovery is less. So, to decrease percent recovery you would need to add more sodium citrate concept: A buffered solution consists of a weak acid and a salt of its conjugate base or a weak base and its conjugate acid. Buffers resist changes in pH by neutralizing added H+ and OH− ions. The pH of a buffered solution depends on the ratio of weak acid and its conjugate base.

q8

In the Pourbaix diagram, the boundary lines between the species domains indicate the transition from one species into another. Along the boundary between an aqueous and a precipitating solid species, equilibrium can exist. The domain boundary between Cu2+ and CuO (at pH of 7) is a vertical line parallel to the y-axis (potential). Above +0.3 V, this line is a constant and does not change regardless of increasing potential. Therefore, the equilibrium between Cu2+ and CuO is independent (unchanging) with respect to applied potentials above +0.3 V. Educational objective: Plots such as Pourbaix diagrams show the relationship between chemical species as a function of given conditions. Boundary lines between domains indicate the transition from one species into another, and can also indicate equilibrium between aqueous and precipitating species. Boundaries parallel to an axis show constant ranges that are independent of changes in the respective axis variable.

question 1

In the [Fe(H2O)6]3+ complex ion, an uncharged water molecule serving as a ligand is a weak Lewis base and can be displaced by the oxalate (C2O42−) anion, a stronger Lewis base. Each C2O42− anion displaces two water molecules and decreases the overall charge of the resulting complex ion by −2. Displacement of water within the complex occurs in a stepwise sequence. In the first step, two water molecules are displaced from [Fe(H2O)6]3+ by a single C2O42− anion to form an intermediate complex ion: [Fe(H2O)6]3+(aq) + C2O42−(aq) → [Fe(H2O)4(C2O4)]+(aq) + 2 H2O(l) You form 2 water molecules bc in the question they ask for 2 covalent bonds. Each covalent bond formation will release one water molecule. And for the overall charge you would just subtract the -2 (charge of oxalate anion from +3 (charge of the solution) In the second and third steps, additional C2O42− anions displace the remaining water ligands to form the final product. concept: When a reaction occurs in a stepwise sequence, the species formed as products in earlier steps and then subsequently consumed as reactants in later steps are intermediate species. Intermediates do not appear in the overall net reaction.

q8

In the conversion of Ge into Ga, the number of protons (atomic number) must decrease by 1. Because the mass number of 68 is unchanged, the loss of 1 proton must then be accompanied by the gain of 1 neutron. Therefore, Ga is generated from Ge by converting a proton to a neutron (as occurs during electron capture). Also think look at the periodic table and realize that Ge has an atomic number of 32 and Ga has an atomic number 31. If Ga is being generated from Ge than the atomic # decreases by 1 which can only happen if an electron is captured. concept: Nuclear conversions may change the atomic number (number of protons) or the mass number (combined number of protons and neutrons). In beta decay, a proton can convert to a neutron (or vice versa), leaving the mass number unchanged.

q53

In the diagram given for the cerium oxide catalyst, the exact roles of the CeO2 and Ce2O3 species are not given. Instead, the function of the catalyst is illustrated by showing four chemical conversions that the catalyst performs. Individual chemical reactions describing these chemical conversions should be in agreement with the diagram. The diagram of the cerium oxide catalyst shows that CO is converted into CO2; therefore, if the reactions are irreversible, the catalyst would NOT function for the reverse reaction (converting CO2 to CO), which is depicted by the reaction CO2 + Ce2O3 → 2 CeO2 + CO In the reverse reaction, CO2 is a reactant and CO is a product, which is not shown in the diagram. concept: Catalytic diagrams use curved reaction arrows to depict the conversion of one chemical species into another through the interaction with a catalyst. These diagrams focus on the function of the catalyst and may not explicitly show intermediate species. Chemical reactions should be consistent with the catalyst function.

Q3

In the experiment described in the question, a strong acid (H2SO4) is added to an ammonia buffer system. The amount of H2SO4 added is much less than the concentration of NH3 and will react with the NH3 to form NH4+ (a weak acid); therefore, the pH will decrease slightly but the ammonia buffer system will resist a large change in pH. concept: Buffer systems counteract H+ and OH− ions. When small amounts of a strong acid or base are added to a buffer solution, the solution pH changes only slightly. Adding strong acid causes the pH to decrease slightly whereas adding a strong base causes the pH to increase slightly.

question 37

In the reaction discussed in this question, elemental aluminum Al(s) replaces hydrogen in HCl to form AlCl3. In turn, the hydrogen atoms combine to form elemental hydrogen gas (as indicated in Table 2), which exists as H2 because elemental hydrogen is most stable as a neutral diatomic molecule. Concept: Single replacement reactions involve the replacement of one type of atom with another in a reaction between a neutral element and a compound such that a new compound and a new neutral element are produced.

question 54

In this question, the given compounds all have σ bonds to a sulfur atom. As a result, the longest bond between sulfur and another element will be formed with the element that has the largest atomic radius. Because Cl has a larger atomic radius than H, N, or F, the S-Cl bond in S2Cl2 will the longest bond with sulfur among the given compounds. Largest atomic radii = longest bond. Smallest atomic radii = shorter bond concept: Covalent sigma bonds are made by sharing electrons through the end-to-end overlap of atomic orbitals. The length of a sigma bond can be estimated as the sum of the atomic radii of the bonded atoms. Atomic radii tend to decrease across a row and increase down a column on the periodic table.

Q7

In this titration, 100 mL of 0.1 M HCl were required to reach the equivalence point. Accordingly, the number of millimoles of HCl required to reach this point is 100 mL×0.1 mmol HClmL =1 0 mmol Because each mole of ammonia gives one equivalent of base and each mole of HCl gives one equivalent of acid, the neutralization reaction in this titration proceeds with a 1:1 molar ratio. Therefore, 10 mmol of HCl were required to neutralize 10 mmol of NH3 present in the unknown ammonia solution.

q55

Ionization energy (IE) in order: 1st IE < 2nd IE < 3rd IE he first, second, and third ionization energies are the energies required to remove the first, second, and third electron, respectively. Because the valence shell is farther away from the positively charged nucleus, electrons in this shell are easier to remove than the inner core electrons. The electrons are removed in order beginning with the electron of highest energy. Magnesium has two electrons in its valence shell (n = 3). Magnesium's third ionization energy is significantly larger because it requires the removal of an electron from the inner core. These electrons are more tightly held than the valence electrons, and therefore require more energy to remove. concept: Ionization energy is the energy required to remove an electron from a ground-state gaseous atom or ion. The first, second, and third ionization energies describe the energy required to remove one, two, or three electrons, beginning with the electron of highest energy. Electrons in the inner core require significantly more energy to remove than valence electrons.

q4

Just had to look at the passage figure 1 of the electronic configuration where you see that its paramagnetic (meaning it had unpaired electrons). In paramagnetic, the electron spins parallel to the magnetic field concept: Hund's rule states that orbital filling maximizes the number of unpaired electrons. Paramagnetic atoms and molecules have unpaired electrons that align parallel to an applied magnetic field; paramagnetic species are weakly attracted to magnets. In contrast, diamagnetic atoms and molecules have no unpaired electrons; these species are repelled by magnets.

Q22

Ka is given and asking for pkb. Find Kb from Ka by substracting it from 14. and that should be the pKb bc pkb = 1.0x10^-# concept: The acid dissociation constant Ka and the base dissociation constant Kb of an acid and its conjugate base are related by the self-ionization constant of water Kw according to the relationship KaKb = Kw. This relationship can also be expressed using a logarithmic approach as pKa + pKb = pKw.

q17

Keq > 1 = spontaneous Keq < 1 = nonspontaneous Keq = 1 = equilibrium concept: For a reaction to be spontaneous in the standard state, ΔG° must be negative. Because ΔG° = −RT ln(Keq) and the natural logarithm of any number greater than 1 is positive, Keq must be greater than 1 for a standard state reaction to be spontaneous. Similarly, a Keq between 0 and 1 corresponds to nonspontaneous reactions, and a Keq equal to 1 corresponds to a reaction at equilibrium in the standard state.

q19

Knew II and VI were right but didn't know III was right As the atoms in the alkaline-earth metal group increase in size moving down the column, the valence electrons occupy an increasingly higher energy level and therefore are less tightly bound. Accordingly, less energy is required to remove the electrons from the valence shell of larger alkaline-earth metals. Consequently, the reactivity of the alkaline-earth metals increases with increasing atomic number (Number III). concept: Group 2 of the periodic table contains the alkaline-earth metals, which are a highly reactive family of metals with only two valence electrons that are readily lost to form cations with an oxidation state of +2. Alkaline-earth metals react with water to form hydrogen gas and either a basic hydroxide or an oxide, and the reactivity increases with increasing atomic number.

q5

Look at image for the conversion concept: Given the mass of a compound sample, the number of moles present in the sample can be determined using its molar mass (the amount of mass present in 1 mole of a substance). Relating a known number of moles of a reactant to a balanced reaction equation, mole ratios can be applied to determine the number of moles of one chemical species relative to the number of moles of another chemical species in the reaction.

question 15

Looked at calcium and phosphate position in periodic table and saw that ca is metal and phosphate is nonmetal so had to be ionic. And again knew it would be polar and not non polar because these two atoms have an electric dipole. Nonpolar covalent bonds form between atoms with a difference in electronegativity no greater than 0.5. These bonds involve equal sharing of electrons, and they frequently form between two atoms of the same type (eg, carbon-carbon bond). Carbon-hydrogen bonds are also considered nonpolar because both atoms have similar electronegativities. No such bonds exist in calcium phosphate. concept: Bonds between atoms are classified generally as ionic, polar covalent, or nonpolar covalent. Ionic bonds involve complete transfer of electrons from an atom with low electronegativity to an atom with high electronegativity. Covalent bonds involve electron sharing, and are considered polar if the sharing is unequal but nonpolar if the sharing is equal.

q59

Looked in the psg it said ""at lower reaction temps compound 2 is major product and at higher rxn temps compound 3 is major product"" so could figure out that compound 2 = kinetic and compound 3 = thermodynamic. But can also look at graph and notice that Compound 2 has a lower energy transition state than Compound 3. Therefore, Compound 2 will form faster and is the kinetic product. Compound 3 has a higher energy transition state, but it is more stable (lower energy) than Compound 2. Therefore, Compound 3 is the thermodynamic product. Educational objective: For a reaction producing more than one product, the product with the lowest energy transition state will form fastest and is the kinetic product. The compound with a higher energy transition state that produces the lowest energy (most stable) structure is the thermodynamic product

q20

Looking at the roman numerals individually: I = this one is explaining the molar mass of (NH4)2CO3 II = this one is talking about the avogadros number. A gram of a sample will contain 1 mole (6.022 × 1023 units) of (NH4)2CO3 molecules III = 96.11 amu of (NH4)2CO3 refers only to a single molecule of (NH4)2CO3, which does not contain the same mass as a mole of (NH4)2CO3 molecules. Amu provides a useful measure for assessing the masses of different atoms concept: The molecular weight is the sum of the atomic mass units contributed from each atom in a formula unit. One mole (6.022 × 1023 formula units) of a given molecular weight equals the molar mass, which is the amount of mass (in grams) contained in 1 mole of the formula units (grams/mole).

q25

Loosely bound electrons (valence electrons) are removed first before any core electrons. Accordingly, removing a core electron takes more energy than does removing a valence electron. On the periodic table, metals generally have a lower first ionization energy than nonmetals, and first ionization energies tend to increase across a row and decrease down a column. The same is true for the second ionization energy except when the second electron being removed is not a valence electron. Group 1 is the wrong ans bc they only have 1 valence electron and the 2nd ionization energy requires the loss of a tightly bound core electron concept: The second ionization energy is the energy required to remove the second of two electrons from an atom. The second ionization energy tends to increase across a period and to decrease down a group; however, ionizations involving core electrons are higher energy than those involving valence electrons.

q19

Manipulation of the the ideal gas law. PV=nRT Boyle's law says that volume and pressure has an inverse relationship: P1V1 = P2V2, P∝1/V PV = k Concept: Boyle's law shows an inverse relationship between the pressure and the volume of a fixed number of molecules of an ideal gas at constant temperature.

q12

Mass % = solute mass/ (solute + solvent mass) x100 concept: Mass percent (% m/m) is defined as the component mass divided by the total mass of the formula unit or mixture expressed as a percentage. The total mass of a solution mixture equals the combined mass of solute and the solvent, and the density of a liquid must be used to find its mass to complete a mass percent calculation.

q54

Metals - less negative electron affinity and harder to add an electron Nonmetals - more negative electrons affinity and easier to add an electron concept: Electron affinity is the change in energy associated with adding an electron to a neutral atom in the gas state. The more negative the electron affinity, the more readily an atom accepts the addition of an electron. Most nonmetals tend to accept an additional electron more readily than most metals.

question 3

Metals tend to lose electrons to form cations because of their low electronegativity and ionization energy whereas nonmetals tend to gain electrons to form anion. For each electron lost, the charge increases by one unit, but for each electron gained, the charge decreases by one unit. Answer is D because the alkaline-earth metals Ca and Mg each have 2 valence electrons, which are easily lost during oxidation in natural systems to form cations with a +2 charge. Answer C is wrong because Ca2+ and Mg2+ cations are produced by the oxidation of elemental Mg and Ca metals, not by reduction. Remember, oxidation is the loss of electrons and reduction is gain of electrons. Concept: Metals tend to lose electrons to form cations whereas nonmetals tend to gain electrons to form anions. The charge of an atom increases by one unit for each electron lost but decreases by one unit for each electron gained. Metal cations with a smaller ionic radius and a higher positive charge are stronger Lewis acids than those with a larger ionic radius and lower charge.

q31

NaI is stated as a halogen homogenous catalyst in the psg which means that it will decrease the activation energy of a reaction by stabilizing the transition state and this causes the rate of the rxn to increase Educational objective: The activation energy of a reaction is the difference in energy between the reactants and the transition state of a reaction. Catalysts lower the activation energy by stabilizing the high-energy transition state

q21

Need to know avogadros law that says that mols (n) is directly proportional to volume. Here the volume is being increased 3x so the mols will also be increased 3x concept: Avogadro's law indicates that at constant temperature and pressure, the number of moles of a gas within a container is directly proportional to the volume of the gas.

q35

Need to know the characteristics of an ideal gas: 1) Kinetic energy increases with higher temperature 2) Collisions btw the molecules are completely elastic (no energy is lost by interactions or friction 3) No attractive or repulsive forces btw the molecules 4) Relatively negligible molecular volume (equal to about 0) concept: An ideal gas is a hypothetical gas consisting of molecules that have no intermolecular attractions or repulsions, no molecular volume, perfectly elastic collisions, and an average kinetic energy that is directly proportional to the gas temperature. The ideal gas concept is used as a model for systems involving real gases.

q10

Need to know what the law of mass action is The law of mass action is usually expressed with respect to the equilibrium constant of a reaction. But when applied to an elementary reaction, the law of mass action states that the rate of a reaction is proportional to the molar amount (concentration or partial pressure) of each reactant raised to the power of its reaction order. Thats why an increase in the ratio of H2/O2 will increase the reaction rate concept: The law of mass action states that the rate of a reaction is proportional to the molar amount of each reaction component raised to the power of its reaction order. For elementary reactions, the reaction order of each species is equal to its stoichiometric coefficient.

q19

Needed to know the combustion reaction. The most common one is the answer Combustion reactions involve the burning of a fuel (often a hydrocarbon, an alcohol, or another flammable organic compound) in the presence of oxygen, often from the air. When the supply of oxygen is plentiful, complete combustion occurs to form carbon dioxide (CO2) and water. concept: Chemical reactions can be categorized broadly into five main types: combination, decomposition, single replacement, double replacement, and combustion. Combustion reactions involve the burning of a fuel in the presence of oxygen to form carbon dioxide and water (and sometimes other oxides).

question 13

One of the main things to realize when doing this question is that MgCO3 doesn't have a charge because Mg has a +2 charge and CO3 has a -2 charge. So when they are together, the charge cancels out. When I do the double replacement, i have to bring that charge back and thats why I would get H2CO3 and not HCO3. After the double replacement, H2CO3 breaks because it is an unstable acid and MgCl2 is an ionic salt so it's stronger. concept: Double replacement reactions involve the exchange of the bonding partners of two reactant compounds, forming two new product compounds. Decomposition reactions involve a single compound breaking down into two or more new compounds. Understanding reaction types and bonding patterns allows the prediction of reaction products.

question 55

Out of all the choices, (K2SO4) contains a metal. Therefore, potassium must form a metal-nonmetal (ionic) bond in the structure in addition to the nonmetal-nonmetal (covalent) bonds present between the nonmetal atoms in the structure. concept: The type of bond formed between two atoms depends on the relative difference in electronegativity between the atoms. Atoms with a large difference in electronegativity (usually a metal and a nonmetal) form ionic bonds. Atoms with a small difference in electronegativity (usually two nonmetals) form covalent bonds.

question 9

Passage based question and had to look at the diagram. (picture) concept: Infrared spectroscopy can be used to detect structural changes within molecules during chemical conversions by monitoring successive sample spectra for the emergence or disappearance of peaks that are unique to a given structural feature of a molecule.

q41

Passage says that the y axis of the graph is equivalent to gibbs free energy. Because reactions with a ΔG° < 0 can occur spontaneously, a redox process with a larger positive E° is more favorable than a redox process with a smaller E°. The passage states that on a Frost diagram, the slope of a line segment joining two species is equal to E° for the couple. As a result, when comparing line segments, the segment with the greatest positive slope is the more favorable reaction. comparing the slope btw ClO4- and ClO3- for both acidic and basic pH, the slope for acidic conditions (pH 0) is greater than the slope for basic conditions (pH 14). Therefore, performing the experiment under acidic conditions will make the reduction of ClO4− more favorable during the reaction. concept: Reactions associated with a larger positive standard reduction potential are more thermodynamically favorable. The standard reduction potential E° for an oxidation-reduction couple can be evaluated graphically from electrochemical data.

q37

Passage states that the slope of a line segment joining two species on a Frost diagram equals the standard reduction potentialE° for the couple. In the eqaution, ClO3- is reduced to HClO2. In the frost diagram, at acidic pH, ClO3 gives you the initial condition and HClO2 gives the final condition. The change in oxidation number between the two species (Δx) equals the number of electrons transferred, and the slope of the line segment between these points equals the standard potential for the reduction. As stated in the passage, the slope of a line segment joining two species on a Frost diagram equals the standard reduction potentialE° for the couple. The change in oxidation number between the two species (Δx) equals the number of electrons transferred, and the slope of the line segment between these points equals the standard potential for the reduction. Substitute the x- and y-coordinate values for the ClO3− and HClO2 points on the Frost diagram into the slope equation above gives: Slope: (5.0−7.4) V⋅e−/ (3−5)e− = −2.4 V⋅e−/−2 e− = +1.2 V concept: The standard reduction potential E° for an oxidation-reduction couple can be evaluated graphically using electrochemical data. The slope of a line segment between two points on an x,y-coordinate system is found by dividing the change in the y-axis values by the change in the x-axis values.

q9

Pauli exclusion principle says that within a subshell you can hold a max of 2 electrons which have opposite spins. These electrons are represented by arrows, and the spin of each electron is indicated by the orientation of the arrow (up or down). The orbitals containing the electrons are often represented by blanks or boxes. As the orbitals are filled, Hund's rule states that electrons remain unpaired until pairing is required to add another electron. Following the Aufbau principle, the Pauli exclusion principle, and Hund's rule, the eight electrons in oxygen must have a configuration of: 1s^2 2s^2 2p^4 concept: An electron configuration accounts for all electrons held within the orbitals of an atom or ion, listed sequentially by shell and subshell in order of increasing energy following the Aufbau principle, Hund's rule, and the Pauli exclusion principle.

Q7

Pauli's exclusion says that paired electrons must be +1/2 and -1/2 concept: According to the Pauli exclusion principle, each orbital within a subshell can hold a maximum of two electrons, but two electrons in the same orbital must have opposite spins. Electron spin is a vector quantity with a magnitude of ½ and a direction that is either spin up (positive) or spin down (negative).

q40

Q asking for moles of electrons and they give you that 6 mol electrons reduce 1 mole CLO3- so stoichiometry Educational objective: The number of moles transferred in a sample taken from a stock solution can be calculated by multiplying the molarity of the stock solution by the sample volume. Molar ratios taken from balanced chemical reactions or electrochemical processes can be used to relate the moles of one species to the moles of another.

q11

Q asking to find Cl- IONS in lanthanum(III) chloride so you can find the molecular formula by knowing that if Lanthanum has a 3+ charge then Cl- has to equal 3- and we know Cl- has a 1- charge so molecular formula is: LaCl3 see image Educational objective: The mole ratio is the whole number proportion of each chemical species in a balanced chemical equation. Molarity is the number of moles of solute per liter of solution. Because molarity expresses concentration in terms of moles, a mole ratio can be used to relate the concentration (molarity) of one species to the concentration of another species in solution.

q38

Q asks abt I2 so just had to find ox# for I in chem equation so it goes from 0 to +5 so its being oxidized which means its a reducing agent Educational objective: A reducing agent is a chemical species that gets oxidized and causes reduction in another atom in the process, whereas an oxidizing agent is a chemical species that gets reduced and causes oxidation in another atom as a result. Oxidation and reduction always occur together.

q57

Q asks which would be MOST reactive with Group 6A and &A nonmetals and the most reactive would be the one iwth the lowest IE cuz its the ""easiest"" one to take an electron off of in comparison to the others so it would be the most reactive one form here to react with teh nonmetals low IE = most reactive w/ nonmetals high IE = least reactive w/ nonmetals Educational objective: The alkali metals are very reactive because of their low ionization energy, large atomic radius, and small electronegativity. Cesium is considered the most reactive metal.

q47

Qstem says under aqueous environment so i looked at the table and saw that enthalpy and entropy are both negative. That means the rxn will only be spontaneous under low temperature and D says the same thing but just the opposite way so thats why its the answer Concept: The Gibbs free energy ΔG equation is used to determine the spontaneity of a system, where negative ΔG is spontaneous and positive ΔG is nonspontaneous. The sign of the change in entropy ΔS for a process determines whether it becomes more or less thermodynamically favorable (ie, spontaneous) as the temperature changes.

q29

Remember anions are larger than cations Ionic radii tend to decrease in size across a period (row) of the periodic table (left to right) and increase moving down a group (column). This trend for metal cations resets and repeats for anions beginning near the division between metals and nonmetals, past which anions tend to preferentially form. Compared to the neutral atom of a given element, its cation will be smaller but its anion will be larger. Losing electrons to form a cation causes the remaining electrons to experience a greater effective nuclear charge Zeff, pulling the electrons closer to the nucleus. Conversely, gaining electrons to form an anion produces greater electronic repulsion and nuclear shielding (lesser Zeff), which pushes electrons farther from the nucleus. concept: Ionic radii decrease in size across a period and increase down a group on the periodic table. This trend occurs for metal cations and then resets and repeats for anions beginning near the division between metals and nonmetals. The use of an isoelectronic series is helpful in comparing cations, anions, and neutral atoms.

q49

Since the graph is given, you just find half of the initial amount and that will give you 1.5x10^5. They are asking about 2 half lives, so you take half of 1.5 and thats .75. If we cared about scientific notification it would be 7.5x10^4 but it doesn't matter here bc we only care about the mins. .75 matches up with the graph around 40 mins and the closest ans is 44. OR after finding the initial amount, match that up with the x axis time and multiply that time with 2 bc they are asking about 2 half lives. 22x2 will give you 44 which is the answer bc thats at 2 half lives concept: The half-life is the time required for the amount of a given radioactive isotope to decrease by half. With each subsequent half-life interval that passes, the amount that remains decreases by half again, becoming exponentially smaller and eventually approaching zero.

q18

Solid < liquid < gas; solid is less dense in water and thats why ice floats. (but for all other materials solid is more dense than liq and liq is more dense than gas) really hot = really flying around = less dense really cold = not flying around = more dense Answer is saying that since the temp of the solution is decreased, the solution becomes more dense = density increases fo the solution and so it will make the object float d=m/v Density can vary slightly with temperature Initially, at 25 °C, the mass-to-volume ratio (density) of the sealed glass bulb exactly matches that of the solution in the cylinder. As the temperature of the solution in the cylinder decreases, the density of the solution increases (same mass occupying a contracted volume). Because the glass bulb acts like an isolated system, its density changes very little compared with that of the cylinder solution. Accordingly, the density of the cooling solution in the cylinder becomes greater than that of the bulb, causing the bulb to begin to float. This is the basis for the function of the Galileo thermometer. Educational objective: The density of a substance is defined as the mass it contains per unit of volume that it occupies. The exact density of a substance can vary slightly with temperature due to minute changes in volume caused by the expansion or contraction of materials. Solid objects float in liquids with a higher comparative density and sink in liquids with a lower comparative density.

question 2

Stronger Lewis bases can displace weaker Lewis bases as ligands within a complex. Stronger Lewis bases tend to be those that have lone pair electrons on atoms with a net charge and/or a lower electronegativity. Charged species tend to be less stable (more reactive) than uncharged species because forming a bond produces a more stable, lower energy state. Likewise, atoms with a lower electronegativity tend to be better Lewis bases because the electrons are held more loosely and any net charge is less stabilized. Both C2O4^2− and H2O have available lone pair electrons on an oxygen atom (same electronegativity aspect), but C2O4^2− is a stronger Lewis base because it also has a negative charge on each of two of the oxygen atoms. This charge comes from the presence of an extra lone pair of electrons on each. Therefore, C2O42− does not have fewer lone pair electrons per oxygen atom than H2O, and this choice does NOT explain why C2O4^2− displaces H2O. Another way to look at it is by looking at the charge for oxalate anion which has a charge of -2. Thats means that oxalate anion has 2 more electrons than water. concept: Stronger Lewis bases (electron pair donors) can displace weaker Lewis bases as ligands within a coordination complex. Lone pair electrons on atoms with a lower electronegativity tend be stronger Lewis bases than those with a higher electronegativity, and charged atoms with lone pair electrons tend to be stronger Lewis bases than comparable uncharged atoms.

question 24

Stronger bonds require more energy to break (dissociate) than weaker bonds. The overall strength of a bond results from the sum of all σ and π bonding contributors. As a result, when comparing bonds involving the same two types of atoms, a triple bond is stronger than a double bond, and a double bond is stronger than a single bond because double and triple bonds are composed of both σ and π bonds. However, if the strengths of the σ bond and π bond contributors are considered separately, π bonds are weaker than σ bonds. The end-to-end orbital overlap in σ bonds is more efficient than the side-to-side orbital overlap in π bonds. This causes σ bonds to exist in a more stable, lower energy state. As a result, breaking a σ bond requires more added energy than breaking a π bond (ie, a σ bond has a greater dissociation energy). concept: Sigma bonds are lower in energy, more stable, and have a greater dissociation energy than π bonds. Although individual π bonds are weaker than σ bonds, double and triple bonds are composed of both σ and π bonds and are therefore stronger overall than a single bond.

question 28

Tetrahedral - sp3, 4 bonds, no lone pairs Trigonal pyramidal - sp3, 3 bonds, one lone pair Trigonal planar - sp2, 3 bonds, no lone pairs Bent - looks like water concept: The geometry of a molecule can be determined according to valence shell electron pair repulsion (VSEPR) theory, which states that the distribution of bonds and lone pairs will minimize repulsion between electrons. To find the geometry of a molecule, count the number of electron domains and determine the electron group geometry, keeping in mind that lone pairs have greater repulsive forces than bonding pairs.

q3

The Heisenberg uncertainty principle states that the exact position and momentum of a particle cannot both be known at the same time. In fact, the uncertainty of position and momentum is inversely proportional: the more accurately an object's position is known, the less accurately its momentum can be known, and vice versa. This assumption is why we dont have the exact position of electrons but instead electron orbitals are now modeled as probability distributions in which electrons are most likely to be found. concept: The Heisenberg uncertainty principle states that the position and momentum of a particle cannot both be precisely known at the same time. The uncertainty of the position and the uncertainty momentum are inversely proportional.

q46

The Zeff is higher for F bc it increases as you move from left to right. When comparing atoms within the same period (row) of the periodic table, this approximation shows that Zeff increases as the atomic number increases, but this approximation does not distinguish between different rows of the periodic table and only shows that Zeff increases as the group (column) number increases. Although 18F and 18O both have two core electrons, 18F has more protons. Therefore, when comparing the effective nuclear charge of 18O to the effective nuclear charge of 18F, the 18F atom has the greater atomic number and higher Zeff concept: The nuclear charge Z (number of protons) influencing a valence electron is diminished by the shielding constant S, which results in an effective nuclear charge Zeff that is less than the full nuclear charge. By setting S equal to the number of core electrons, Zeff can be estimated as Zeff = Z − S.

question 20

The acid with the smallest pKa (largestKa) is the strongest acid and will yield the solution with the lowest pH. Of the organic acids compared in the table above, citric acid is the most acidic (largest Ka, smallest pKa) and will produce a solution with the lowest (most acidic) pH. Concept: Acid strength increases as the acid ionization constant Ka increases, or as the acid pKa decreases. As a result, the pH of a solution at a given concentration decreases as the pKa decreases.

q9

The amount of product formed in a reaction depends on the equilibrium constant of the reaction. This constant is not changed by the presence or absence of a catalyst, which affects only the rate of the reaction. Platinum acts as a catalyst and affects the rate but not the equilibrium of the reaction. Therefore, both engines would produce the same amount of water in one cycle. concept: The amount of product formed in a reaction is dependent on the equilibrium constant of the reaction. This constant is not changed by a catalyst, which only changes the rate at which equilibrium is achieved.

question 17

The arrangement of the periodic table indicates charges that atoms are likely to adopt. Metals in the first column of the periodic table, including potassium, ionize by losing one electron; potassium has a charge of +1. Metals in the second column, such as calcium, lose two electrons and have a +2 charge. The reaction in the passage shows that one calcium ion is neutralized by one oxalate ion, and three potassium ions are neutralized by one citrate ion. Therefore, oxalate must have a charge of −2 and citrate a charge of −3 to neutralize their respective cations. concept: Ionic bonds form when positive charges neutralize negative charges. One ion may carry multiple charges, and must be neutralized by the same number of opposite charges.

q32

The chemical behavior of an atom is determined by the configuration of electrons in its valence shell, as this number determines how many electrons must be gained or lost to achieve stability. Elements in the same column of the periodic table have the same configuration of valence electrons, and therefore have similar chemical properties. Zirconium is in the same column as hafnium (Hf), and therefore any isotope of hafnium will have similar chemical properties to any isotope of zirconium. concept: Chemical behavior is governed largely by the number of electrons in an atom's valence shell. Atoms with the same number of valence electrons (same column of the periodic table) have similar chemical properties.

question 4

The compounds H2O, H2S, H2Se, and H2Te each contain bonds with hydrogen. The difference among these compounds is which of the Group 16 elements (oxygen group, or chalcogens) is bonded to hydrogen. Because electronegativity tends to decrease moving down a group (column) on the periodic table, tellurium is the least electronegative of the four chalcogens. As a result, the electronegativity difference between hydrogen and tellurium in the H−Te bond is the smallest of these four hydrogen-chalcogen bonds, and therefore it has the smallest dipole moment. concept: The difference in electronegativity between two covalently bonded atoms is proportional to (and indicative of) the magnitude of the dipole moment along the bond between the atoms. Larger differences in electronegativity invoke larger dipole moments between atoms.

question 5

The coordination number of a metal complex refers to the number of coordinate bonds formed between the central metal ion and its nearest neighboring atoms. When all of these nearest neighboring atoms are from separate molecules or ions, the number of ligands will equal the coordination number. However, if two or more of these nearest neighboring atoms are joined to the same coordinating ligand unit, then the number ligands will not equal the coordination number. In both cases, the number of nearest neighboring atoms and coordinate bonds is unchanged, but the number of ligand units is different. In coordination complexes, the ligand works as a lewis base and the metal center acts as a lewis acid. concept: Coordinate covalent bonds are formed between two atoms when one atom donates both shared electrons. Such bonds are often formed between electron-poor metal ions and electron-rich atoms in ligands in a complex, and the coordination number of the complex refers to the number of coordinate bonds formed with the metal ion.

q1

The easiest way to go about this is realizing that Equation 1 corresponds to the slope equation. ln Ka = is the y −ΔH°/R = is m (R is the gas constant) To figure out if the rxn is endothermic and exothermic, just solve for ΔH using the equation without any numbers bc only signs are relevant here. −m= −ΔH°/R mR= + ΔH° m is negative bc the slope of the graph is negative and since the enthalpy/R is also negative, at the end you get a positive enthalpy which means endothermic. Extra info: An endothermic reaction has a positive ΔH° because heat is added to the system (heat absorbed from the surroundings) whereas an exothermic reaction has a negative ΔH° because heat is produced by and lost from the system (heat released into the surroundings). concept: An endothermic reaction has a positive ΔH° and absorbs heat from the surroundings whereas an exothermic reaction has a negative ΔH° and releases heat into the surroundings.

question 32

The energy of the orbitals determines which wavelengths of light can be absorbed. Some ligands cause greater differences in the energy of the d orbitals than other ligands, resulting in the absorption of different wavelengths of light. These wavelengths are usually in the visible spectrum. When heme binds O2, the nature of the O2 ligand changes the energy of iron's d orbitals. This energy change causes heme to absorb blue-green light and reflect red light. Also, color is not only associated by transition metals so since heme is exhibiting the color that means it has to do with d orbitals. concept: The nature of the ligands in a coordination sphere causes the metal's d orbitals to have different energies. This energy difference determines the wavelength of light absorbed by the bonds. The wavelength of light that is reflected is often in the visible region of the electromagnetic spectrum.

Q14

The equilibrium for this process is quantified by the acid dissociation constant Ka. Strong acids have an equilibrium favoring the products (the release of H+ and A−) and ionize essentially 100% in aqueous solutions, resulting in a large acid dissociation constant (Ka > 1). Weak acids have an equilibrium favoring HA, the non-ionized acid, and ionize only to a small extent, resulting in a small acid dissociation constant (Ka < 1). As stated in the question, aspirin is a weak acid so the Ka value for aspirin is expected to be small (Ka< 1). This is consistent with the low solubility reported for aspirin. A strong acid with Ka > 1 would ionize to a large extent and would produce highly water-soluble ions, as seen for the ionic sodium salt of aspirin. The low solubility of the neutral aspirin acid indicates a low extent of ionization in solution. Concept: Strong acids ionize essentially 100% in aqueous solutions, resulting in a large acid dissociation constant (Ka > 1), whereas weak acids ionize only to a small extent, resulting in a small acid dissociation constant (Ka < 1).

q3

The experiment in the psg tells you that ΔH° is positive (4911R) and ΔS° is positive (5.4R) after solving for them. Have to know the chart for spontaneous and nonspontaneous rxns for gibbs free energy and that would show you that the reaction is nonspontaneous at low temperatures (Number II) but spontaneous at high temperatures (Number III). As such, the reaction is endothermic and is not spontaneous at all temperatures (Number I). Educational objective: A spontaneous process does not require a continuous input of additional energy. The change in Gibbs free energy (ΔG°) is evaluated as ΔG° = ΔH° − TΔS° and is used to identify whether a process is spontaneous (ΔG° ˂ 0), nonspontaneous (ΔG° > 0), or at equilibrium (ΔG° = 0). Processes in which both changes in entropy ΔS° and enthalpy ΔH° are positive are spontaneous at high temperatures and nonspontaneous at low temperatures.

question 22

The formula to find the answer was given in the passage concept: The equation ΔG = −kRT log(Q) describes the free energy (ΔG) of a system, where R is the gas constant, T is the absolute temperature, and Q is the reaction quotient (concentration ratio) related to the system. For systems involving hydrogen ion concentrations, the pH correlates directly to [H+] by the relationship pH = −log[H+] (or [H+] = 10−pH), and ΔG for H+ transport is related to differences in pH.

question 50

The side chain of threonine contains a highly polar O-H bond, and this gives threonine a strong, permanent dipole that can participate in dipole-dipole interactions bc its polar. Alternatively, the side chain of valine contains only nonpolar hydrocarbon bonds that lack a strong permanent dipole. This means that valine cannot participate in dipole-dipole interactions. concept: Dipole-dipole interactions are a type of noncovalent interaction that occurs between neighboring polar molecules with polar bonds and a net dipole moment. The partial charges within nearby dipoles experience a mutual attraction and align in such a way that partial negative charges orient toward partial positive charges.

question 20

The key part of this question is that it's a weak base. Think about bronsted bases - they are H+ acceptors. So, that means that potassium citrate is going to accept the H+ ions and that would cause a decrease in the hydronium (H3O) in the urine. Also the q stem says that what can the researchers do to decrease the uric ACID in urine. If we are trying to decrease an acid we are obviously going to take away H+ ions to make it less acidic. Thats another way to get to the answer. Concept: The pH of a solution is equal to the negative logarithm of the hydronium ion concentration. It is related to hydroxide concentration by pH = 14 − pOH and can be increased by adding hydroxide ions or removing hydronium ions.

question 33

The metal ion Fe2+ is positively charged whereas the donor atoms (nitrogen) are neutral, resulting in a net charge of +2 for the complex. Unlike ionic bonds, the donor atoms do not give electrons to the positively charged metal. Instead counterions often surround the complex in solution to balance the metal's positive charge. Also, looking at the diagram you can see Fe2+ has a charge and that is the coordination complex meaning the compound would be charged. concept: Coordinate covalent bonds are a special bond between a central atom and its ligands. The number of coordinate bonds indicates the coordination number and the ligands provide bonding electrons, which interact with the metal's d orbitals to form the coordinate covalent bond.

question 44

The only freely rotating bonds is single bonds concept: Covalent bonds are formed by sharing electrons between atoms through the overlap of atomic orbitals in an end-to-end or side-to-side configuration. Multiple bonds formed by s and p orbitals consist of one σ bond and one or more π bonds. Free rotation around a bond is possible with σ bonds (single bonds) but is prevented by the structure of π bonds (double and triple bonds).

question 45

The overall dissociation energy of a bond (the energy required to break the bond) results from the sum of all σ and π bonding contributors. Therefore, the overall bond dissociation energy and bond strength tend to decrease (relative to a triple bond) as the number of π bonds decreases (Number I). Because of the shapes of the orbitals involved, bond length tends to increase (relative to a triple bond) as the number of π bonds decreases (Number II). Because σ bonds are made by an end-to-end orbital overlap, free rotation around the bond between the two σ-bonded atoms can be achieved while maintaining the orbital overlap. Therefore, decreasing the number of π bonds also decreases the rigidity of the bond between the atoms (Number III). concept: Multiple covalent bonds formed by s and p orbitals consist of one σ bond (end-to-end orbital overlap) and one or more π bonds (side-to-side p orbital overlap). Decreasing the number of π bonds between two atoms increases bond length, decreases bond rigidity, and decreases the overall bond dissociation energy and strength.

Question 43

The overall energy of a bond is the sum of the energies of all σ and π bonding contributors. When comparing single, double, and triple covalent bonds involving the same two types of atoms, the overall bond dissociation energy (the energy required to break the bond) tends to increase (relative to a single bond) with each additional π bond. This is due in part to increased electron density between the atoms that participate in multiple bonds. Out of the molecules given, only carbon monoxide has a C≡O triple bond (one σ bond and two π bonds); therefore, it has the highest bond dissociation energy of the carbon-oxygen bonds shown. concept: Covalent bonds are formed by sharing electrons between atoms through the overlap of atomic orbitals in an end-to-end or side-to-side configuration. Multiple bonds formed by s and p orbitals consist of one σ bond and one or more π bonds, and the overall bond dissociation energy of a bond tends to increase relative to a single bond with each additional π bond.

question 21

The passage states gastric acid has a pH of 1.0. Therefore, the concentration of hydrogen ions can be determined by the expression: [H+] = 10−1 = 0.1 M = 0.1 mmol/mL The number of mmol of H+ in 10.0 mL of gastric acid can be determined as follows: (0.1 mmol/mL)(10.0 mL) = 1.0 mmol of H+

question 6

The question is asking about equivalence point. Equivalence point is in the middle of 2 pka values and they give us the pka values so thats why the answer is b The number of pka values depends on how many H that compound has concept: In a titration of a weak polyprotic acid and a strong base, the pH at a stoichiometric equivalence point is determined by the pKa values of the acid and the conjugate base species present in the equilibrium reaction.

q46

The question is asking under which environment would you get the highest amount of product. Looking at the graph, you see that the Y axis is activation energy. The lower the activation energy, the faster the reactants are turning into products. Since aqueous environment has the lowest Ea, you will get the highest amount of products there. But also you have to know that kinetic vs thermodynamic control matters. When the conditions are such that a reaction is irreversible (eg, low temperature), it is said to be under kinetic control, and the most favorable product is the one whose pathway has the lowest activation energy. Unless the reactions have had time to complete, the fastest reaction will yield the most product. When a reaction occurs under conditions such that it is reversible (eg, high temperatures), it is said to be under thermodynamic control. Under these conditions, all products form readily and the most stable product will be favored (ie, the most negative ΔG), as it is least likely to undergo the reverse reaction. Here, all the environments were irreversible aka under kinetic control so you can look at the graph and compare. concept: Irreversible reactions tend to be under kinetic control, which favors products whose pathways have the lowest activation energy. Reversible reactions tend to be under thermodynamic control, which favors the most stable product.

q32

The question says that the temperature increased more quickly, indicating that the reaction rate was higher when the catalyst was ground to a fine powder. The smaller particles of MnO2 provided more surface area to be exposed to H2O2 molecules, which allowed the reaction to proceed at a faster rate. Always remember that whenever its a q about grinding something to a fine powder they most likely talking about surface area concept: A heterogeneous catalyst is in a different phase than the reactants whereas a homogeneous catalyst is in the same phase as the reactants. Heterogeneous catalysts involve the adsorption of reactants onto the surface of the catalyst and provide higher reaction rates with increased surface area.

q17

The representative elements (also called the main group elements) are the elements in Groups 1-2 and Groups 13-18 on the periodic table. These elements are so called because they are the most abundant elements in nature and provide a wide representation of the elemental and chemical characteristics found in natural systems. concept: The representative elements (main group elements) are the elements in Groups 1-2 (1A-2A) and Groups 13-18 (3A-8A) that are contained in the s-block and the p-block, respectively, of the periodic table.

Q23

The small Ka value for HCN indicates that the non-ionized form is favored, and it is a weak acid. Evaluation of the [H+] gives [H+] = 1 × 10−6 M. This [H+] is much less than the HCN concentration (1 × 10−3 M) and confirms that only a small percentage of the dissolved HCN molecules in solution are ionized (consistent with a weak acid). Concept: In aqueous solutions, strong acids ionize essentially 100%, giving large values of Ka and high [H+]. Weak acids ionize only to a small extent, giving small values of Ka and [H+] that are much lower than the concentration of the non-ionized acid.

question 22

The type of bond that forms between two atoms depends on the relative difference in electronegativity between the atoms. A large difference in electronegativity promotes ionic bond formation, but a small difference promotes covalent bond formation. As a result, ionic bonds generally form between a metal and a nonmetal, whereas covalent bonds generally form between two nonmetals; however, there are some exceptions because the ionic character of a bond exists on a continuum. Therefore, in chemical bond formation, covalent bonds are generally formed by electron sharing between two atoms with a small or moderate electronegativity difference. concept: The type of bond formed between two atoms depends on the relative difference in electronegativity between them. Atoms with a large difference in electronegativity (usually a metal and a nonmetal) form ionic bonds. Atoms with a small difference in electronegativity (usually two nonmetals) form covalent bonds.

question 49

The weakest of the van der Waals forces are London dispersion forces, which are also known as induced dipole-induced dipole interactions. These interactions occur between two molecules whose proximity to each other induces a mutual attraction via the creation of a weak, temporary dipole moment. Within a nonpolar molecule, the average electron distribution does not concentrate around any particular nucleus sufficiently enough to produce a permanent dipole moment. However, when two nonpolar molecules come near each other, the electron cloud of one molecule can momentarily shift and form a localized, slightly positive region that then induces a slightly negative region in a neighboring molecule. Consequently, instantaneous dipoles with weak mutual attraction are formed momentarily. In region IV of the protein in Figure 2, there are only nonpolar alkyl side chains (no permanent dipoles). Therefore, the primary interactions in region IV are just London dispersion forces (the weakest type). concept: London dispersion forces, also called induced dipole-induced dipole interactions, are the weakest of the noncovalent van der Waals forces. These interactions occur between two atoms or molecules that are near enough to each other for distortions in the electron cloud to induce weak instantaneous dipoles.

Q11

They are asking for the mol ratio of the products. Easiest way to do this would be to look at the equation given and see how mane moles of each product is formed. 1 mol of copper (II) acetylsalicylate and 2 mols of sodium sulfate is formed. Thats why the answer is 1/2 concept: The coefficients in a balanced chemical reaction indicate the stoichiometric proportions by which each chemical combines during the reaction. Mole ratios derived from these coefficients can be used as conversion factors in chemical reaction calculations to relate the molar amounts of compounds in a reaction.

q4

They are asking for volume of H2 gas from the liters of k2MnO4. you have to perform conversion (see image). concept: Given the volume of a solution, its molarity, and a balanced reaction, mole ratios can be determined that relate the moles of a reactant present in the solution to the moles of a product produced from the solution. For gaseous products, the number of moles produced can be related to the molar volume using 22.4 L/mol, the volume 1 mole of gas occupies at STP.

question 56

They are different compounds bc they are not one compound from the periodic table. By looking at the compounds, you can see that SO3^(-2) has a -2 charge which means that it has 2 more electrons and a different bonding configuration than SO3 by itself. concept: Identical chemical species must have the same elemental composition, the same number of electrons, and the same orbital bonding configuration. The net charge of a structure is equal to the sum of all charges present.

question 14

They give you two reactions and you just had to figure out the overall reaction and cancel out the electrons concept: Redox reactions are fully balanced when the same number of each type of atom and the same net charge are present on both sides of the reaction equation. For redox processes expressed as two half-reactions, the net reaction is found by adding the two half-reactions (or multiples of them) together so that the electron terms cancel.

Q2

To find the neutrons, you subtract the atomic number from the mass number. Electrons = atomic number and since this one has a charge of -2, you add +2 to the number of electrons concept: Isotopes are specified by their mass number (the number of protons and neutrons in the nucleus). The number of neutrons in an isotope can be determined by subtracting the element's atomic number (number of protons) from the mass number. An atom in its elemental state is neutral (equal number of protons and electrons) but acquires a net charge of +1 for each electron lost and −1 for each electron gained during ionization.

Question 1

To find the second equation, flip the rxn and add it to the overall rxn (make sure to cancel whatever cancels) In the question, the first reaction is the overall reaction. The second reaction is step 1. They are asking to find the next step that will give you the overall rxn. You can go ans choice by ans choice and add step 1 to the answer choices to see if it gives you the overall reaction. Adding choice B gives the overall reaction. For the reaction to go to completion, the two oxygen atoms must combine with two O2 molecules to form two ozone molecules (2O + 2O2→ 2O3) in the second step. This yields the net reaction 3O2 → 2O3. Concept: According to the law of conservation of mass, atoms are neither created nor destroyed in a chemical reaction. The same number of each type of atom must be present on both the left and right sides of a reaction arrow. Multistep reactions that go to completion must consume all intermediates produced before or during the final step.

Q4

To neutralize H2PO4 fully completely you need 2 equivalents of NaOH -> one OH- from NaOH for each H+ concept: Polyprotic acids have more than one proton that can dissociate in water. The titration curve of a polyprotic acid will show as many plateaus and steep climbs as the number of protons in the acid. These represent the buffer regions and equivalence points in a titration, respectively.

question 12

To separate a solution mixture of CuF2 and BaF2 by precipitating CuF2, Cu2+ ions can be introduced into the solution. The increased Cu2+ concentration disturbs the product side of the solubility equilibrium and causes it to shift toward the reactants (undissolved salt). Therefore, adding Cu(NO3)2 as a source of Cu2+ ions will selectively cause the CuF2 in the mixture to precipitate. concept: In solutions containing two salts that have an ion in common, the common ion supplied by one salt produces a common ion effect that shifts the equilibrium of the other salt to the left and decreases its solubility. The common ion effect is a specific scenario involving Le Châtelier's principle that can be used to separate compound mixtures.

Q6

To visualize the pH change and that the titration is complete, you have to use an indicator that has an endpoint near the pH of the equivalence point. The best indicator for a given titration is one that has a pH range that corresponds most closely to the pH of the equivalence point. concept: Indicators are added to a titrated solution during an acid-base titration to visually detect when a titration is complete. The best indicator for a titration is one that has an endpoint (color change) pH range nearest to the pH of the equivalence point.

question 13

When an ionic compound AX2 is placed in water, the composite ions A2+ and X− will dissociate to some extent into the solution and establish an equilibrium in which the rate of dissolution (forward reaction) equals the rate of precipitation (reverse reaction). This equilibrium position is indicated by the solubility product constant Ksp, which provides a measure of the solubility of a compound at a given temperature. Because the Ksp of CuF2 (1.6 × 10−6) is smaller than the Ksp of BaF2 (3.0 × 10−6), a saturated aqueous mixture of CuF2 and BaF2 will have fewer moles of CuF2 present in the solution and less CuF2 to remove from the solution during a separation (Choice C). The Ksp of a compound is constant at a given temperature, and the concentrations of dissolved species cannot increase beyond that consistent with the Ksp. This means that if [A2+] increases, [X−] must decrease so that the value of the Ksp remains unchanged. If the Ksp is small, then small changes to [A2+] or [X−] have a much larger impact than would be the case if the Ksp were large. Therefore, CuF2 (smaller Ksp) is more responsive than BaF2 (larger Ksp) to small changes in common ion concentrations, and fully precipitating BaF2 would require a greater number of common ions to be added to the solution. Although adding F− as the common ion would cause CuF2 to precipitate, it would also cause BaF2 to precipitate. The fact that adding F− precipitates CuF2 does not indicate why precipitating CuF2 is a better choice than precipitating BaF2. concept: The solubility of a compound is indicated by the solubility product constant Ksp, and the Ksp values of two compounds can inform which compound to selectively precipitate during the separation of a compound mixture using the common ion effect.

q50

When comparing similar molecular structures, molecules with a greater number of hydroxyl groups (-OH groups) have more sites capable of hydrogen bonding. As the number of -OH groups increases, more energy is required to overcome the additional attractions from hydrogen bonding and the boiling point rises. Accordingly, only 1,2,3-propanetriol (3 -OH groups) has a higher boiling point than 1,3-propanediol (2 -OH groups). Structures with a greater extent of intermolecular forces have higher boiling points because more energy is required to overcome the attractions between molecules. Among similar structures, the boiling point tends to increase as the number of hydroxyl groups increases due to a greater extent of hydrogen bonding interactions.

q57

You just need to look at the graph and compare the delta G. Compound 3 had a higher neg difference so it is exergonic. ΔG = (Gibbs free energy)products − (Gibbs free energy)reactants concept: The change in Gibbs free energy (ΔG) during a reaction is evaluated as ΔG = (Gibbs free energy)products − (Gibbs free energy)reactants. If the energy of the products is less than the energy of the reactants (−ΔG), the reaction is exergonic (energy is released), but in the opposite case (+ΔG), the reaction is endergonic (energy is consumed).

q6

When two reactants are present, the rate law is expressed generally by the equation: Rate = k[A]^m[B]^n where k is a constant, m and n are the order of the respective reactants, and (m + n) is the overall reaction order. To determine the actual rate law, the values of m and n must be found by comparing the given reactant concentrations and initial rates. Comparing Trial 2 to Trial 1, [A] is left unchanged but [B] is doubled relative to Trial 1, causing the reaction rate to double. Therefore, the rate has a first-order dependence upon [B], and n = 1. Similarly, when comparing Trial 3 to Trial 1, [A] is doubled but [B] is left unchanged, and the rate doubles; so, the reaction rate also has a first-order dependence upon [A], and m = 1. This result is further verified by comparing Trial 4 with Trial 1; doubling both [A] and [B] gives a rate four times greater than that seen for Trial 1. Therefore, the overall reaction rate must be a second-order expression (m + n = 2) involving variable terms for both [A] and [B].

q7

Whenever you compare the trails for one substrate, you have to make sure for those trials the other substrate's concentration is constant. Ex: for A we compare trial 3 and 4 bc at 3 and 4 trial B is constant. Look at the picture. That helps the most Comparing Trial 2 to Trial 1, [A] is left unchanged but [B] is doubled and the reaction rate quadruples. This means that the rate has second-order dependence on [B] (n = 2). Comparing Trial 3 to Trial 4, [A] is tripled but [B] is left unchanged. The rate triples, indicating that the reaction rate has first-order dependence on [A] (m = 1). Therefore, the overall reaction is third-order (m + n = 3), and the rate law can be written as: Rate = k[A][B]^2 ⇒ k= Rate/[A][B]^2 Educational objective: The reaction order of each reactant in a chemical process can be determined by comparing kinetic data at different concentrations, from which the rate law can be written. The rate constant k can then be evaluated from the rate law.

q14

You had to convert mols of Ca3(PO4)3 to grams of LaPO4 using the reactions to do mol to mol conversion. See image concept: The mass of a product can be calculated given a particular amount of reactant by first calculating the number of moles of product formed using a reactant-to-product mole ratio derived from the balanced chemical equation. The moles of product can then be converted to grams using the molar mass of the product.

question 53

You had to draw out the lewis structure and when you draw it out you can tell that neither of those compounds are correct. (Choice A) Compound 1 has 10 valence electrons around one oxygen atom. Bonding that involves only s and p orbitals and follows the octet rule will have no more than 8 valence electrons around any given atom. (Choice B) Although Compound 2 has 8 valence electrons around each atom and follows the octet rule, the overall structure has a total of 20 valence electrons and has a net charge. Atoms from Group 16 each contribute only 6 valence electrons. Therefore, SO2 should have a total of only 18 valence electrons and no net charge. concept: Lewis structures symbolically depict chemical compounds by using chemical symbols to represent atoms and dots placed around the symbols to represent the valence electrons and their bonding configuration. Dots are arranged to give each atom a full valence shell of electrons following either the octet rule or a valid exception.x

q2

You had to use the electronic configuration given in the passage and you see the last 3 electrons are from the 5d and 6s shells so since the charge here is 3+, the electrons would be lose from those shells concept: The electron configuration of an atom describes the placement of all the electrons within the atom. During ionization, electrons are removed from the orbital with the greatest principal quantum number.

q44

You had to use the graph in the passage to find the time that corresponds to 1.5 half lives. Find 50% of sample which gives the time of 110 mins. And to find the time at 1.5 half lives you multiply 110 by 1.5 which gives you 165 mins. Match up the x-axis with 165 and that will give you the % of sample remaining at 1.5 half lives (35%). Multiple that 35% with 200 to give you the amount of sample in milimercuries at 1.5 half lives concept: The activity of a radioisotope (the number of decay events per unit of time) is proportional to the amount of the decaying radioisotope present, which decreases by 50% for each radioisotope half-life that passes.

q6

You have to find the K+ ions in each solution and add the mols of both together and divide by the L of solution which is 1 L here bc .8L + .2 L = 1 L. (see image) concept: The number of moles transferred in a sample taken from a stock solution can be calculated by multiplying the molarity of the stock solution by the sample volume. When two samples from different stock solutions are mixed, the molarity of the sample mixture equals the total number of moles transferred divided by the total mixture volume.

q27

You have to find the volume using ideal gas law and then use the density equation to find the density. The answer is given in mL so you have to convert the volume you get from ideal gas to mL bc you get it in liters. concept: An ideal gas is a hypothetical gas with well-behaved (idealized) characteristics used to make a simplified model to predict the behavior of real gases. The behavior of a gas can be described by the ideal gas law. Using a calculated gas volume and its mass, the gas density (mass per unit volume) can be evaluated.

q4

You have to use the gibbs free energy equation that relates to Keq: ∆G°=−RT ln Keq The Keq here is the Ka because thats the equilibrium constant for an acid dissociation. The equation is: Ka = [H+][A−]/[HA] Plug that Ka equation in to the ∆G° equation: ∆G°=−RT ln([H+][A−]/[HA]) For this you can tell that ∆G and pH are directly proportional pH of a weak acid at a given temperature is found by the relationship: pH=−log[H+] According to the data from Table 1, as the temperature increases, the pH decreases ([H+] increases), and Ka increases. This corresponds to a decrease in ΔG°. Because of the negative value H+ and delta G is inversely related. Therefore, ΔG° at pH 2.0 is lower than ΔG° at pH 2.6. You can also use this table see the Ka trend corresponding to pH and from the equation delta G and Ka are inversely related, so you can get the answer that way. Concept: The change in Gibbs free energy ΔG° for a reaction at a given temperature is related to the equilibrium constant Keq by the relationship ΔG° = −RT ln Keq. The values of pH and ΔG° are related by the acid dissociation constant Ka, a special type of equilibrium constant.

question 23

atoms that can hydrogen bond are: FON ( fluorine, oxygen, nitrogen) concept: Hydrogen atoms covalently bonded with nitrogen, oxygen, or fluorine atoms form polar bonds that yield dipoles, which exhibit significant noncovalent dipole-dipole attractions known as hydrogen bonding.

question 34

concept: A complex ion consists of a central atom, typically a metal ion, and its surrounding ligands, which form coordinate covalent bonds to the metal. The number of coordinate bonds to the central atom is the coordination number.

Q25

concept: A reducing agent causes a reduction to occur in another compound by giving electrons to the other compound. Therefore, a reducing agent itself undergoes oxidation due to the loss of electrons. The oxidation state accounts for electrons gained or lost by a particular atom.

question 9

concept: Given the mass of a compound sample, the number of moles present in the sample can be determined using its molar mass. Relating a known number of moles of a reactant to a balanced reaction equation, mole ratios can be applied to determine the stoichiometry of one chemical species relative to another species in a reaction.

question 2

concept: Molarity, the number of moles of solute per liter of solution, describes the concentration of a solution. Stoichiometric mole ratios and molarity can be used to assess the moles in an aliquot of solution.

Q27

concept: Osmosis is the diffusion of solvent across semi-permeable membrane from a solution of lower solute concentration into a solution of higher solute concentration until the solution concentrations equalize. During osmosis, solvent diffusion imparts an osmotic pressure that is calculated by Π = iMRT, which is the product of the van 't Hoff factor, the solute molar concentration, the ideal gas constant, and the absolute temperature.

question 26

concept: Resonance structures are Lewis dot structures of a given molecule in which electrons have been delocalized. Acceptable resonance structures move only electrons and generally only to adjacent atoms, keep the number of valence electrons constant, obey the octet rule, do not break sigma bonds, and keep the overall charge of the molecule constant.

q9

concept: The density, percent mass, and molecular weight of a substance can be used to calculate the concentration (molarity) of a solution. The units must be consistent when carrying out these stoichiometric calculations.

q11

concept: The energy of a photon is given by Ephoton=hc/λ. For a molecule or atom to transition from ground to an excited state, it must absorb a single photon with energy exactly equal to the energy difference between the two states.

question 19

concept: Theoretical yield is the maximum amount of product that can be formed in a reaction. It is governed by the limiting reactant, which is determined by the balanced equation and the molar amounts of each reactant available.

q13

decay problem concept: Radioactive decay stabilizes atomic nuclei by releasing energetic particles. Alpha emission releases a helium nucleus (two protons and two neutrons), decreasing the atomic weight by 4 and the atomic number by 2. Beta minus emission releases a nuclear electron during the conversion of a neutron into a proton, increasing the atomic number by 1. Gamma emission is the release of a photon and has no effect on atomic mass or atomic number.

question 39

easier way to do 2/27: 2x10^0/2.7x10^1 = 2/3 x10^(-1) = .6x10^-1 = .06 concept: The limiting reactant is the reactant that is completely consumed first in a chemical reaction. It limits and determines the amount of product that can be produced based on the stoichiometric ratios of the chemical reaction.

Q3

gamma emission releases a high energy photon without altering the elements atomic mass or atomic number concept: Radioactivity is the spontaneous emission of radiation (photons or particles) from an unstable atomic nucleus. The three most common types of radioactive decay are alpha decay (ejection of an alpha particle), beta decay (ejection of a nuclear electron or positron), and gamma emission (release of a high-energy photon).

q1

in coordination chemistry, the center metal ion is the lewis acid and the ligands are the lewis base concept: Lewis bases are electron donors, whereas Lewis acids are electron acceptors. Lewis definitions differ from the Arrhenius definitions, in which an acid is an H+ donor and a base is an OH− donor, and from Brønsted-Lowry definitions, in which an acid is an H+ donor and a base is an H+ acceptor.

question 11

its not square planar because those have 2 lone pairs concept: Hybrid orbitals form when two or more atomic orbitals (s, p, d, f) combine into new orbitals. The number of hybrid orbitals (electron-dense areas) around an atom dictates its hybridization and electron geometry, whereas only the orientation of bonds around the central atom determines the molecular geometry (shape) of a molecule.

q12

periodic trend concept: Atomic radii are governed by the distribution of electrons surrounding the atom's nucleus. They quickly increase from top to bottom in a group and gradually increase from right to left across a period on the periodic table.

question 47

question asks to find which regions have H- bonding and ionic bonding. Look at pic Concept: Ionic bonds are ion-ion interactions between two charged species. When two oppositely charged species are within proximity of each other, they will form an ionic bond via electrostatic attraction. Hydrogen bonding is a special type of noncovalent dipole-dipole interaction involving species with partial charges.

Q1

since they are asking about the spectra for S and Se, you just had to match up the individual S and Se lines to get the total spectra From the wavelength of a line spectra, you can tell the colors, energy and frequency concept: Each element emits photons only at specific wavelengths to give a unique emission line spectrum corresponding to its electron configuration.

q14

since they are talking about molecules, you had to use avogadros number = 6.0x10^23 concept: The density of a substance varies slightly with temperature and is defined as the mass of the substance per unit of volume that it occupies. Avogadro's number (NA = 6.022 × 1023) of formula units equals one mole, and the molar mass of a compound is the amount of mass (in grams) contained in 1 mole of formula units (grams/mole).

q21

talking about ionization energy Because halogens have a lower first ionization energy than their neighboring noble gases, and because Cl is larger and farther down the halogen group than F, the first ionization of Cl will require the smallest amount of energy of the given reactions. concept: The first ionization energy is the energy required to remove the first, most loosely bound valence electron from a neutral atom. The first ionization energy tends to increase with increasing atomic number moving across a period and decrease moving down a group on the periodic table.

question 4

the passage said that oxytocin is stable at 4.5 pH so the buffer closest to that range can be used as a substitute concept: A buffered solution consists of a mixture of a weak acid and a salt of its conjugate base or a mixture of a weak base and its conjugate acid. When the concentrations of the weak acid (or weak base) and its conjugate are equal, pH = pKa. The best buffer for maintaining a given pH should have a pKa that is within one unit of the desired pH.

q6

the psg says that when zinc is not present the arms form coordinate bonds with Gd prohibiting it from binding to water = decrease relaxivity. This means when zinc is present, the arms will not bind gd and gd can bind to water to increase relaxivity. The q says that at time B there was an increase in relaxivity meaning zinc is present and bound to Gd concept: Contrast agents increase MRI signals by increasing the relaxivity of water molecules. These agents can be used to monitor biological processes if their mechanisms involve biochemical changes associated with those processes.

q12

the q is asking what is the concentration ratio btw the stock solution and the standard solution. To find the concentration ratio: [stock solution]/[standard solution] The passage gives the stock solution as .01M (1x10^-2) The question tells you to pick the standard solution with the lowest nonzero and for the graph thats .0001M (1x10^-4) Now just solve using the equation above and you will get the concentration ratio concept: A standard solution can be made by diluting a particular amount of a stock solution. Because the number of solute particles stays the same, the ratio of concentrations can be calculated to determine how much smaller the concentration of the standard solution is compared to the stock solution.

q33

the question says ATOMIC NUMBER BUT THE GRAPH SHOWS ATOMIC WEIGHT so you had to match it up on the periodic table concept: Nuclear fission is the splitting of a large atomic nucleus into multiple smaller nuclei. The atomic weights (protons and neutrons) of resulting nuclei can be measured and correlated with their most likely atomic numbers (number of protons).

q48

used reasoning to solve this one but can also use Arrhenius equation but can make it easier to understand by getting rid of negative exponent by bringing that whole term to denominator. Increasing the temp of the rxn gives a smaller value of the exponential term. Because it is in the denominator, a smaller exponential term gives an increased value for k (Number I). Decreasing the activation energy Ea also gives a smaller exponential term, and therefore k increases exponentially with decreasing Ea (Number II). Conversely, k decreases exponentially with increasing Ea. According to the rate law equation, the rate of a given reaction is directly proportional to the rate constant. Therefore, the reaction rate also decreases exponentially with increasing Ea (Number III). (Number IV) Catalysts lower the value of Ea of reactions, and therefore an uncatalyzed reaction has a higher Ea than the catalyzed reaction. Because uncatalyzed reactions have a higher Ea, the reaction has a decreased rate constant with respect to the rate constant of the catalyzed reaction. Educational objective: The Arrhenius equation describes the relationship between the rate constant and the temperature and activation energy of a reaction. Both the rate constant and the rate of reaction decay exponentially with increasing activation energy and increase exponentially with increasing temperature. Because catalysts lower the activation energy of a reaction, the rate constant is greater for a catalyzed reaction than for an uncatalyzed reaction.

Question 17

you are trying to find the amount of an acid so that means you have to dissolve it in base first and then titrate it with an acid and an indicator Concept: Acid-base neutralization reactions can be used in titrations with an indicator to determine the concentration of an acidic (or basic) analyte. By assessing the equivalence point of the neutralization, the volume of titrant required can be correlated to the amount of an analyte present in solution. Acids or bases may also aid the solubility of nonpolar analytes by forming more soluble ionic salts.


Kaugnay na mga set ng pag-aaral

Ch 2 Quiz The Middle American Realm

View Set

Module 11 Psychological Disorders

View Set

Niche tourism. Unit 6. English for international tourism. Intermediate

View Set

Food Science 2 (Heat Processing)

View Set

Gastrointestinal Disorders 2nd set

View Set

Cost: Are these fixed, variable or semi-variable?

View Set

Chapter 4 Computer Information Systems

View Set

Managerial accounting chapter 15

View Set

PrepU: Chapter 17: Newborn Transitioning

View Set